PT A Expl
PT A Expl
PrepTest A
Explained
„ 2006 Kaplan, Inc.
All rights reserved. No part of this book may be reproduced in any form, by photostat, microfilm, xerography, or any other
means, or incorporated into any information retrieval system, electronic or mechanical, without the written permission of
Kaplan, Inc. LSAT is a registered trademark of the Law School Admission Council.
Section 1: Logical Reasoning
1
PrepTest A Explained
(C) The author says nothing about what happens to could be: workers spend too much time on undeserved
politicians who ignore the economic interests of large breaks. The Keyword “since” tells us that we’re going
groups of people. This answer choice is not at all to get some evidence for the argument, and indeed,
supported by the stimulus. the manager claims that with a reduction in the number
(E) Whatever effectiveness a political interest group of workers assigned to each manager, the managers
may have at its inception, it is excessive growth, not will be able to supervise the workers more closely, so
the mere passage of time, which will cause a loss of as to keep the workers from wasting time on
effectiveness, according to this author. unauthorized breaks. The keyword “therefore” signals
the manager’s conclusion: productivity will soon
4. (E) Flaw increase.
An assumption question focuses attention on the
Subtle shifts in scope can lead to reasoning errors. relationship between the evidence and the conclusion.
The safety inspector is concerned about a recent Here, we are looking for an answer choice that tells us
increase in the number of laboratory samples of the that closer supervision of workers will in fact lead to
rabies virus being sent through the university delivery increased productivity overall. (E) gives us exactly what
service, and claims that that use of the service must we are looking for—the gain in productivity due to fewer
be limited. The biologist utterly fails to address the unauthorized breaks will exceed the lost productivity
recent change in circumstances. He argues that the due to closer supervision.
lack of accidents over the past 20 years proves that When we apply the Kaplan Denial Test to this answer
this use of the university delivery service poses no choice, we confirm that this is the correct answer. If
danger. (E) pinpoints the scope shift between the two the productivity gains did not exceed the productivity
speakers. losses, then the conclusion would not follow from the
(A) brings up an irrelevant consideration. The reason evidence.
why there has been an increase in the use of the (A) and (D) are outside the scope of the argument. The
university delivery service to ship laboratory samples conclusion is a prediction that the impending reduction
of the rabies virus says nothing about the potential in the number of employees assigned to each manager
danger involved. will result in an increase in productivity. Identification of
(B) is a 180. The biologist does focus on the issue of the best (A) or most efficient (D) way to improve
rabies virus samples. productivity will not affect whether the predicted
increase in productivity will occur.
(C) The safety inspector’s argument does not depend
on a change in the “hazardousness” of the rabies (B) The manager provides no indication that the
samples being sent through the university delivery excessive break time is a recent problem. This answer
system; he is concerned with the recent increase in the choice makes an irrelevant comparison.
number of samples going through the system. You (C) is another answer choice that is outside the scope
can’t commit flawed reasoning by failing to consider of the argument. Even if there were other ways to
something outside the scope of the argument you are increase productivity, that says nothing about whether
attempting to rebut. closer super vision of the workers will increase
(D) misses the point at issue. The safety inspector productivity.
does not propose eliminating the transportation of
rabies virus samples; he just wants there to be some 6. (B) Point at Issue
limits on this use of the university delivery system.
Use the Kaplan Decision Tree to identify the point at
There is no logical necessity for the biologist to prove
issue.
that the university needs to deliver the rabies virus
samples from one place to another. Tom begins the argument with the observation that the
large disparity between executive salary and the
5. (E) Assumption average worker’s salary justifies public resentment
over the size of executives’ salary.
Use Keywords to distinguish evidence from the
Martha says the resentment is not justified, because
conclusion.
executives deserve to earn more than the average
The manager has identified “one reason why” worker. According to Martha, executives create wealth
(evidence) productivity in the office is lower than it by taking risks and making decisions, tasks most
2
Section 1: Logical Reasoning
people avoid, and so they should be generously that executives should be generously rewarded, is
rewarded. supported by evidence: executives take risks and make
Tom then clarifies that he didn’t say that people resent decisions that most people choose to avoid.
executives for making much more money than the (C) Tom does not accuse Mar tha of making
average worker. Rather, people resent the atypically contradictory statements. He does not address her
large disparity between executive and average worker statements other than to suggest they are irrelevant to
incomes. We consider each answer choice in turn, and his point.
apply the Kaplan Decision Tree as we go: (E) Tom does not dispute Martha’s evidence for her
(A) Tom claims that in this country, executives make claim that a “large” disparity is warranted.
approximately 85 times more than the average
worker’s salary. Martha says nothing about that 8. (A) Inference
statistic. No disagreement here.
The most modest answer choice is often a valid
(B) Tom says that public resentment of executive
inference.
salaries is justified. Martha says that such resentment
is not justified. They each have an opinion on the The stimulus tells us that A. robustus bones contained
subject, and they disagree, so we have found our point a lower ratio of strontium to calcium than did bones
at issue, and the right answer. For the record, from H. erectus. Further, the lower the ratio, the more
(C) Martha says that a generous wage differential is meat the hominid ate. Finally, we are told that H.
necessary, but Tom offers no opinion on the matter. erectus is known to have been a meat eater. Answer
Not the point at issue. choice (A) makes the very modest claim that the diet
of A. robustus included at least some meat. Given what
(D) Martha clearly states that executives deserve
we have been told, A. robustus must have eaten more
higher salaries than average workers. Tom expresses
meat than H. erectus, and so answer choice (A) must
no opinion on the matter, and could very well agree
be true.
with Martha. No point at issue here.
(B) and (C) Don’t fall into the trap of assuming that
(E) Neither speaker offers an opinion as to whether
just because two things are correlated, one must have
executives in this country create as much wealth as do
caused the other. Nothing in the stimulus tells us
those in other countries. Not the point at issue.
exactly how diet and strontium–calcium ratios are
related, so we have no way of knowing whether the
7. (D) Method of Argument
meat consumed by H. erectus contained more
The correct answer choice for a Method of Argument strontium than that consumed by A. robustus (B) or
question must match the stimulus point for point. whether the diet of H. erectus contained more calcium
than that of A. robustus (C).
In response to Martha’s critique, Tom clarifies that the
(D) Again with the diet! Not only do we not have any
resentment is not due to the existence of a large
basis for tying the calcium and strontium levels in
disparity per se, but rather, the resentment stems from
these hominids to anything in their diets, but also,
the excessive size of the disparity. If the problem is the
whether or not the strontium-to-calcium ratio is a
size rather than the existence of the disparity, then
fraction greater than or less than one makes absolutely
Martha’s point is irrelevant to the argument. Answer
no difference to the information we have been given in
choice (D) describes the argument quite nicely.
this stimulus.
(A) Tom neither tacitly nor explicitly abandons one of
(E) The stimulus neither says nor implies anything
his original premises in response to Martha’s claim.
about the fossilization process affecting the ratio of
He does not introduce a new line of argument
strontium to calcium. This answer choice goes far
defending his original conclusion.
beyond the scope of the stimulus.
(B) How many ways is this one wrong? Martha makes
no generalization about wealth creation. Tom’s new 9. (B) Main Point
evidence (the average size of the disparity between
executive and average salaries in other countries) bears Use the “One Sentence Test” to identify the main
no relation whatsoever to Martha’s statement about the conclusion of an argument.
creation of wealth. He does not disagree with Martha’s
Sometimes the LSAT writers make you work with really
claim about wealth creation. Martha’s “generalization,”
dumb arguments. Don’t get distracted by how bad the
3
PrepTest A Explained
argument is—stay focused instead on the job at hand. during card games. But we are also told that the judge,
Here, we need to identify the main conclusion. despite his exemplary card playing, has been convicted
So in one sentence, what does this letter writer want of criminal assault. So which is the “real” Judge
us to believe? Mosston? The fair and just card player, or the convicted
thug? These two facts alone give us reason to suspect
Judge Mosston should not be forced to resign, (B).
that maybe there is more to Judge Mosston than how
A Main Point stimulus will always include evidence that he plays cards. (C) makes that point quite nicely.
supports that main conclusion. Here, the writer relates
(A) Neither refraining from criminally assaulting
his personal observation that as a card player, Judge
another person nor refraining from cheating at cards
Mosston has always treated his fellow card players
are duties “specific to legal professionals,” and so
fairly. The claim that Judge Mosston is fair (at least
there can be no confusion of those particular duties
when it comes to cards) is supported by the writer’s
with the responsibilities of private citizens.
personal observation, but ultimately, the writer is not
trying to convince us that the judge is someone we (B) The problem with this argument is whether Judge
should trust in a game of poker. This claim, therefore, Mosston’s behavior while playing cards is evidence
is evidence, not the main conclusion. that supports keeping this convicted criminal on the
bench; there is no issue raised by any supposed
The writer also claims that the system cannot afford to
distinction between “fair” and “just.”
lose any judges who are fair and just. This claim is
unsupported. Unsupported claims will never be the (D) Don’t be distracted by a mischaracterization of the
main point of an argument. issues. The writer mentions the card playing not as a
way of minimizing the seriousness of the assault, but
The claim that Judge Mosston should not be forced to
rather, in support of the claim that Judge Mosston is
resign, however, is supported by these other claims,
the type of “just and fair” judge our judicial system
and does not itself support any other claim. That
cannot afford to lose.
makes this claim the author’s main point.
(E) For this answer choice to be correct, we’d need to
(A) The writer assumes that a person who is fair when
see some evidence of popular opinion supporting the
it comes to playing cards is “fair and just” in other
author’s argument. Here, the apparent popular opinion
contexts. To the extent that the writer implies Judge
is that Judge Mosston should be forced to resign,
Mosston is a fair judge, it is to support the claim that
which is exactly the opposite of what this writer
he should not be forced to resign. Not the conclusion
concludes.
of the argument.
(C) The writer is not concerned about judges in 11. (A) Assumption
general, nor is he concerned with ranking important
qualities in a judge. Don’t be too quick to eliminate obvious assumptions.
(D) is far broader than the stimulus. The letter writer According to this author, it is essential to build a space
obviously believes that the assault conviction is not station where astronauts can live for long periods of
sufficient grounds for ending Judge Mosston’s judicial time. Why? So that when there are missions to explore
career. Nothing in the stimulus prevents this writer Mars, we’ll have the medical knowledge about
from arguing that a conviction for criminal assault astronauts’ capacity to live in space for an extended
should, for example, constitute sufficient grounds for time. (A) draws our attention to the fact that this
firing a kindergarten teacher. information is needed only if people, and not robots,
(E) The author makes no claim to being the “most carry out the exploration. Confirm this answer choice
objective” assessor of the judge’s fairness. This by using the Kaplan Denial Test (if robots were used to
answer choice brings in an irrelevant comparison. explore Mars, we would have no need of information
about human capacity for living in space for extended
10. (C) Flaw periods of time—the argument falls apart), and we can
confidently move on to the next question on Test Day.
When a writer tries to support a generalization with a For the record:
limited sampling of data, think about the flaw of
(B) There is no suggestion in this argument that
representativeness.
ordinary humans will be the ones exploring Mars. Any
Here, the writer bases his entire argument about Judge differences between astronauts and the rest of us tells
Mosston’s fairness on his observations of the judge us nothing about whether the medical information
4
Section 1: Logical Reasoning
obtained from the operation of a manned space station than other patients doesn’t necessarily tell us anything
is necessary for those missions. about the specific group we are interested in: heart
(C) Outside the scope. The author makes no surgery patients.
guarantees that the information obtained from (E) If many patients (were they patients about to
astronauts on the space station will somehow prevent undergo heart surgery?) receive a placebo instead of
unforeseen medical problems from arising in a future real vitamin E, that tells us nothing about why doctors
mission to Mars. The author is only concerned with might recommend real vitamin E prior to heart surgery.
obtaining information that would allow scientists to
work within the limits of human capacity for living in 13. (D) Method of Argument
space when they plan the mission.
Being able to distinguish new evidence from an
(D) is even further outside the scope of the argument.
attack on existing evidence can get you points on
While the author may indeed envision many future
Test Day.
missions to fully explore our solar system, she does
not speak about those hopes, dreams or plans in this The architect claims that cleaning a stone building with
argument. water is more damaging to the building than leaving the
(E) 180. If the author were to assume that living in building dirty, and so stone buildings must be left dirty
space for an extended period of time presents if they are to last as long as possible. The engineer
insurmountable medical problems, there would be no clearly disagrees with the architect, and brings new
point in building the space station. The author clearly evidence to the argument: it is possible to clean a
imagines that any such medical problems could be stone building using mild chemicals instead of water,
identified and resolved. and the chemicals will not damage the stone the way
that water can. (D) is a lawyerly way of saying that the
12. (A) Paradox engineer has brought up new evidence to refute the
architect’s argument, which is exactly what the
Identification of the two seemingly incompatible engineer does.
observations is crucial in answering a Paradox (A) The engineer does not dispute the architect’s claim
question. that using water to clean a stone building can cause
The mystery we need to solve: patients who are given irreparable damage to the building, nor does the
vitamin E before heart surgery are less susceptible to engineer dispute the architect’s claim that dirt is less
postoperative complications, although as soon as six damaging to a stone building than trying to clean the
hours after surgery, they do no better than patients building with water.
who were not given vitamin E before heart surgery. So (B) Since the engineer’s only claim is that the architect
why bother? Answer choice (A) tells us that those first has failed to consider relevant information, she cannot
six hours after surgery are critical, and thus justifies possibly be attempting to show that the architect’s
giving patients vitamin E before heart surgery. claims are internally inconsistent.
(B) Whatever happens to patients after the six-hour (C) The LSAT writers love to offer answer choices
time period in which vitamin E makes a difference can’t relating to “consequences” of arguments. It is rare,
possibly explain the recommendation to take vitamin E. however, for a stimulus to include evidence of anything
(C) assumes without any support in the stimulus that that might result from (or in other words, be a
a return to normal heart function ends the six-hour “consequence” of) any particular argument. Here, the
period during which vitamin E makes a difference for engineer offers no evidence that anything in particular
heart patients. If the assumption were justified, this will occur if the stone buildings are left dirty.
answer choice deepens the mystery. The fact that (E) The engineer brings in entirely new evidence; she
some patients regain their normal heart function in does not use the architect’s evidence at all in her
less than six hours makes the benefit of taking vitamin argument.
E appear even more elusive.
(D) Like (C), this answer choice says nothing about
why doctors might recommend vitamin E to their heart
surgery patients despite the lack of long-term benefit.
Moreover, evidence that certain categories of patients
are less likely to develop postoperative complications
5
PrepTest A Explained
To quickly locate the parallel argument, compare the Treat a Principle/Strengthen question the same as
most distinctive aspect of the stimulus argument you’d treat an ordinary Strengthen question.
with the answer choices.
To paraphrase the argument, some species of whales
In the stimulus, the speaker identifies three possible are not endangered. Even so, current whale-hunting
candidates for the culture that could have been technology makes it impossible to enforce a ban on the
responsible for the inscriptions, and eliminates two of hunting of endangered species of whales without
the three candidates in order to conclude that of the banning the hunting of all whales. Hunting of
three known cultures, only the third culture could have endangered whale species should be banned.
made the inscriptions. Conclusion: all whale hunting should be banned.
Comparing the conclusion of the stimulus to the As it stands, the conclusion does not necessarily follow
conclusions in the choices eliminates answer choice from the evidence. While it may be impossible to
(B) because in that argument, two of the three enforce a justified ban on the hunting of endangered
candidates could file a claim. whale species unless a ban on all whale hunting is
Comparing the conclusion also eliminates answer enacted, we still don’t know whether we should ban all
choice (D) because in that argument, two out of the whale hunting.
speaker’s three friends are identified as having Answer choice (B) fills a gap in the argument, by telling
particular characteristics. us that if a certain activity ought to be prohibited (e.g.,
We can also use the conclusion to eliminate answer hunting endangered whale species), then any activity
choice (E) because that conclusion does not identify that interferes with the enforcement of that activity
anybody in particular as the one who must have (e.g., any whale hunting) should be prohibited as well.
committed a particular act. The remaining two answer (A) is a clear 180. We are trying to support the
choices, however, pass the conclusion comparison at argument that all whale hunting, even of non
first glance. endangered species, should be prohibited.
The method of argument used in the stimulus is (C) The anti-whaling activist expresses no specific
relatively easy to identify, so we can look for the salient objection to the whalers’ adoption of highly mechanized
elements (two out of three possibilities are eliminated) technology, and does not suggest that this technology
in order to find the parallel argument. is “new.” This answer choice goes beyond the scope of
(A) does precisely what we are looking for. There are the argument.
three suspects in custody. The burglar must have worn (D) This principle does not go far enough to constitute
size nine shoes, but two of the three suspects could support for the activist’s argument. The activist is not
not have worn size nine shoes, and so the only suspect trying to convince us that a ban on all whaling is
in custody who could have been the burglar is the third “permissible”; she argues that the right thing to do
one. For the record: would be to enact a ban on all whale hunting.
(C) appears to follow the pattern of the stimulus, but (E) The activist makes no distinction between
there’s a very subtle twist: the three “suspects” who commercial and noncommercial whale-hunting. The
could have designed the Ultra 180 are not necessarily potential distinction in the argument is between the
the only three potential designers. They are just three hunting of endangered species of whales versus the
well-known designers who worked for the manufacturer hunting of species of whales that are not endangered.
of the Ultra 180 during the appropriate time frame. The The activist believes that all whale hunting should be
argument does not rule out the possibility that a very banned, regardless of whether the hunted species of
patient, but previously unknown designer might have whale is endangered.
designed the Ultra 180.
16. (A) Weaken the Argument
6
Section 1: Logical Reasoning
have been a spy for England during the time he served embassy was exactly the same period during which the
in the French embassy there. The evidence? English spy was sending information [as in (E)], it is somewhat
records show that a spy in the embassy foiled at least more likely that Bruno was in fact the spy.
two assassination plots against Queen Elizabeth. The (A) adds nothing of significance to the argument. The
spy is identified as the only clergyman working at the stimulus tells us that there was one clergyman working
embassy at that time. Bruno had been ordained a at the embassy. Whatever the clergyman’s duties were
member of the clergy long before he started working at says nothing about whether the clergyman was the spy.
the French embassy.
(B) If the pope were condemning Bruno for having been
At first glance, the argument looks pretty good. Bruno a spy, that would strengthen the argument, but that’s
had been ordained as a clergyman, and the only not what this answer choice offers. The pope’s
clergyman working at the embassy was identified as the condemnation of Bruno for his philosophical ideas says
spy. But this being the LSAT, we know that there is an nothing about whether or not Bruno was a spy.
assumption hiding in that argument, and if we can
(C) This answer choice says Bruno was treated the
weaken the assumption, we can weaken the argument.
same as all the rest of the diplomatic staff at the
The speaker assumes that once Bruno was ordained, he
embassy, and like everybody else, was introduced to
continued to be a clergyman. (A) pokes a huge hole in
English aristocrats by the French ambassador. To
that assumption. If Bruno had been excommunicated
strengthen the argument, we need something that
before he started working at the embassy and no longer
distinguishes him from the rest of the embassy staff,
dressed or functioned as a clergyman, there’s a good
since he, unlike the rest of the staff, is purported to
chance that he was not the clergyman working at the
have been a spy.
embassy. Do we know for sure that he was not the
clergyman at the embassy? No, because he was (D) actually weakens the argument. If his
ordained before he started working there. But (A) tells us contemporaries knew Bruno as a teacher (and not a
that perhaps Bruno was not the clergyman/spy, which clergyman), then it is less likely that he was the
makes our conclusion a little less likely to be valid. clergyman/spy.
(B) With at least two assassination attempts against
18. (A) Method of Argument
Queen Elizabeth being foiled, it would not be surprising
for there to be a high degree of tension at the embassy. Identifying the type of evidence used to support the
But that says nothing about whether or not Bruno was conclusion can be crucial to identification of the
the spy. method of argument.
(C) Even if French were not Bruno’s “best” language,
The argument begins with the claim that the current
that does not necessarily mean that he could not have
lack of certainty in some scientific theories is due to
known enough French to have sent the occasional
scientific progress, not anything inherently wrong with
message. Not a weakener.
the theories. Why? The author explains that decades of
(D) One would expect a good spy to be able to gain the research have produced many new discoveries, and as
trust of those around him. That the French ambassador a general rule, when new facts come to light in a
trusted Bruno would be entirely consistent with Bruno par ticular field, that field’s theories will require
having been the spy. adjustment. (A) describes exactly what is happening in
(E) General information about “well-educated” clergy the argument: first the author describes a situation,
during the Renaissance doesn’t tell us anything certain which is then explained as consistent with a general
about what Bruno did at the embassy, and gives us no pattern.
reason to doubt that Bruno was the spy. (B) This author finds nothing unacceptable about the
flux in theories that results from the discovery of new
17. (E) Strengthen the Argument evidence.
The correct answer for a Strengthen question may not (C) 180. This author specifically denies one possible
be sufficient to make the argument invulnerable. explanation for the unsettled state of certain scientific
theories. The two explanations are not equally probable
This question uses the same stimulus as Question 16, according to the author.
so we are still concerned with the issue of whether
(D) No law of nature is cited in the argument. No
Bruno was the spy in the French embassy. If it were
particular kind of change is identified.
true that the period when Bruno worked at the
7
PrepTest A Explained
(E) The author never claims that the lack of certainty time were naturally left-handed Boldavian children
(or anything else) is an intended outcome of the forced to use their right hand to fit in socially).
decades of research that produced the new (A) seems outside the scope of the stimulus at first
information that caused the flux in theories. glance—how would gender differences in life expectancy
explain the discrepancy? But the end of the answer
19. (E) Inference choice ties the new information to the stimulus quite
nicely. If Boldavian men are more likely than Boldavian
Where, as here, the stimulus to an Inference question
women to be left-handed, and the men die earlier, it
is based on an argument, the correct answer choice
would not be surprising for the proportion of left-handed
is usually an assumption—a claim that “must be
Boldavians to diminish as the population ages.
true” for the argument to be valid.
(B) deepens the mystery. If left-handed people are no
The author claims that because all art has political more likely than right-handed people to be involved in
implications, all art criticism is political. The author accidents, that eliminates another possible
then concludes that an art critic who ignores the explanation for the discrepancy between older and
political connotations and tries to evaluate a work of younger Boldavians, with respect to the proportion of
art solely based on aesthetics must, by necessity, left-handed individuals.
endorse the artist’s politics. But even if we accept that
(C) Someone who is ambidextrous would not be
all art criticism is political, it does not necessarily
classified as being left-handed. A high value placed on
follow that a critic who attempts to avoid political
ambidexterity might influence the propor tion of
criticism of a work of art must agree with the artist’s
ambidextrous individuals, but it would not make a
politics. That is the assumption of the argument,
difference in the proportion of left-handed individuals
described in (E). Try the Kaplan Denial Test, and you
in the Boldavian population.
can confirm that (E) must be true for this argument to
be valid. If an aesthetic critique of a work of art could (D) This answer choice invites us to confuse absolute
result in a rejection of the artist’s political statement, numbers with percentages. A declining birthrate would
the argument falls apart. reduce the number of left-handed Boldavians over time,
but the stimulus is all about proportions, and so a
(A) No claim is made about a politically minded critic’s
decline in the population wouldn’t affect the
interest in the aesthetics of art.
proportions at issue in this question.
(B) No claim is made about critics who are “politically
(E) Information about the ceiling on the proportion of
naïve.”
left-handed Boldavians tells us nothing about why the
(C) According to the author, all art is political. The propor tion has declined so precipitously as the
stimulus makes no distinction between art that makes population has aged.
an overt social or political statement and art that
makes no overt social or political statement. 21. (C) Flaw
(D) The author does not assign any relative value to art
criticism that is overtly political versus art criticism that A claim about what is generally true cannot be
attempts to avoid politics. disproved by a counterexample.
According to Eugenia, reliable testing shows that
20. (A) Paradox Koolair brand refrigerators last longer than other
brands, and on average (“on the whole”) require fewer
Don’t be too quick to dismiss an answer choice that
repairs than other brands. Neil’s bad experience with a
seems outside the scope of the stimulus—it may very
Koolair refrigerator, while unfor tunate, does not
well resolve the paradox.
disprove Eugenia’s claim. His example would have
The mystery posed by this question is why the contradicted her claim if she’d said that all Koolair
propor tion of left-handed Boldavians drops so refrigerators require fewer repairs than other brands,
significantly as the population ages. The LSAT writers but she never said that. (C) makes that very point. One
eliminate the two most obvious explanations: the lemon doesn’t necessarily mean that the brand as a
proportion of Boldavians who are born left-handed has whole is poorly made, nor does Neil provide any other
not changed, and neither have Boldavian attitudes evidence to dispute the validity of the product testing
toward left-handedness (meaning that at no relevant Eugenia references.
8
Section 1: Logical Reasoning
(A) There is no equivocation in the meaning of “quality” options. This is a good argument, and a good argument
as between the two speakers. Both use the number of can never be parallel to a flawed argument.
repairs as a measure of quality. (E) Now we are back to a recommendation of choosing
(B) Neil does offer evidence to support his argument— one option over another (butter versus olive oil). The
he relates his personal experience with a Koolair evidence to support the recommendation is not that
refrigerator. olive oil is better than using nothing, but rather, olive oil
(D) Neil does not claim to be an authority on is what European chefs use.
refrigerator quality. He uses his personal experience,
not any supposed expertise, as the basis for his 23. (E) Weaken the Argument
argument.
When the question stem is long and/or unusual in its
(E) is a tempting wrong answer choice. Neil does format, slow down and look for Keywords that tell you
assume that his experience with a Koolair refrigerator what to do.
is representative of the group of Koolair refrigerators
as a whole. However, this answer choice does not The most prominent clue in this question stem is the
identify the flaw of overgeneralizing based on a known phrase “call into question,” a typical indicator of a
characteristic of one member of a group; it describes Weaken the Argument question. Specifically, we are
the flaw of overgeneralizing based on a known looking for the answer choice most weakened by the
characteristic of all members of a group. Neil does not critic’s claim, assuming that the answer choice was
purport to have knowledge about the quality of every suppor ted by the claims made by the health
single Koolair refrigerator, so this answer choice does association.
not correctly describe the flaw in his reasoning. The health association notes that certain studies
showed that most volunteers who used to eat meat
22. (D) Parallel Reasoning (Flaw) were able to adopt a vegetarian diet. The critic’s
response points to the flaw of representativeness: the
Parallel Reasoning/Flaw questions aren’t always volunteers were already interested in tr ying a
hard. vegetarian diet, and despite that interest, many of
“Choose our product, because it’s better than using them failed in their efforts. The critic’s point? Evidence
nothing,” is this advertisement in a nutshell. Is the that some people were able to switch to a vegetarian
product better than its competitors? One thinks not, if diet doesn’t mean that anybody could do it. Answer
this is the best ad they can come up with. That’s the choice (E) makes the claim that most people could
flaw. So we are looking for a recommendation based on succeed in adopting a vegetarian diet, if they were told
a comparison between following the recommendation by their doctors to do so. That conclusion is
and doing nothing. A quick scan of the answer choices undermined by evidence that the studies were not
brings us to (D): Choose graphite as a lock lubricant, representative.
because it’s better than leaving your lock unlubricated. (A) The critic does not dispute the potential health
For the record: benefits of switching to a vegetarian diet.
(A) The recommendation is based on a comparison of (B) This is the critic’s point—people who are favorably
two possible options: add the salt before the water disposed toward trying a vegetarian diet are more likely
boils, or add the salt after the water begins to boil. This to succeed than people who are not already favorably
is not a choice between following the recommendation disposed.
and doing nothing. (C) The critic is not concerned with the number of
(B) This recommendation is actually based on a people who have adopted a vegetarian diet in recent
comparison between the recommended bank and its years—his point is that the study results are not
competitors. The comparison may not be particularly necessarily representative of the general population.
compelling, but it’s not a choice between using the (D) The critic does not address the feelings of those
bank and using no bank. who tried to adopt a vegetarian diet yet failed to do so.
(C) Another recommendation that is based on a
comparison between competitors. This time, the
comparison is based on trials by a consumer
magazine, which addressed the durability and safety of
the tires, qualities that are highly relevant in comparing
9
PrepTest A Explained
Most of the time, the LSAT writers ask for the The correct answer to a Paradox question adds a
necessary assumption, and the Denial Test works to piece of evidence that harmonizes seemingly contrary
confirm the correct answer. However, the Denial Test information.
may not be helpful when the question asks you to
Why should the seller of an especially desirable item at
identify an assumption that would be sufficient to
an auction be particularly interested in the protection
justify the conclusion. In that case, plug your
offered by the setting of a minimum price for a
preferred answer choice into the argument to confirm
successful bid? The usual justification for setting such
that it will justify the conclusion.
a reserve price for an item is that it protects the seller
More than half of the full-time jobs for unskilled from buyers who make a token bid, gambling that
workers in Centerville are provided by the town’s shoe nobody else will bid on the object. The right answer
factory. Does that necessarily mean that if the shoe choice will tell us that there might be an incentive for a
factory closes down, more than half of the unskilled prospective buyer to “gamble” that nobody else will bid
residents of Centerville who used to hold a full-time job on a particularly desirable object. (E) does just that: if
will be unemployed? Note the scope shift between the prospective buyers tend to believe that only an
location of the jobs in the evidence, and the location of unreasonably high bid will be successful, they might
the worker’s homes in the conclusion. not bother bidding at all, leaving an opportunity for a
The conclusion follows only if more than half of the buyer who makes a token bid to be the high bidder.
unskilled residents of Centerville who work full-time are (A) deepens the mystery. If the high bidder is stuck
employed at the shoe factory. Answer choice (D) is with the object, and cannot defer to the next-highest
good enough to guarantee that happens. Note that the bidder, the seller is guaranteed whatever that high bid
Denial Test will not confirm (D) as the correct answer— is. The high bidder would not have an incentive to defer
the argument could still hold if a few of the workers at to a lower bidder unless he’d made a bid in excess of
the Centerville shoe factory are residents of Edgewater the reasonable value of the object, which would be
or some other town. But (D) is good enough to good for the seller (who would have no need for the
guarantee that the conclusion is valid, and so it’s the “protection” offered by setting a reserve price).
correct answer. (B) Whether the unsuccessful bidders are ever
(A) and (B) The argument is concerned only with identified publicly tells us nothing about the
unskilled workers who are employed full-time. General relationship between the reserve price and protection
information about the proportion of employed versus of the seller.
unemployed residents (A) or skilled versus unskilled (C) If the reserve price for any object (not just the most
workers (B) is not good enough the guarantee the desirable ones) did not ensure a profit to the seller, it
validity of the conclusion. would not offer the seller any meaningful protection.
(C) The conclusion says nothing about skilled workers. (D) If prospective buyers could identify other
A higher proportion of unskilled to skilled workers in prospective buyers, they would presumably be less
the factory will not guarantee that the number of likely to gamble by making a token bid, and the seller
unskilled, full-time shoe factory workers is greater than of an extremely desirable object would have less, not
half of Centerville’s population of unskilled, full-time more need of the protection offered by a reserve price.
workers.
(E) The stimulus compares the Centerville shoe factory
to all other businesses in town. Whether or not there
are other factories makes no difference to the validity
of the conclusion.
10
Section 2: Reading Comprehension
11
PrepTest A Explained
(A) is outside the scope of the passage. The passage (C) 180. The Church would not recognize a clandestine
never addresses whether parents were more likely to marriage as described in the second paragraph, but
bow to the dictates of the Church than were their the marriage would be legally valid.
children. (D) Another 180. The Church would not recognize a
(B) The author never discusses parental preferences clandestine marriage. The passage draws no
as between the two types of marriage contracts. connection between a clandestine marriage and a
(C) The author says nothing about parental concerns marriage arranged for reasons of economic
for Church versus legal recognition of any particular expediency.
marriage. (E) De futuro contracts are not mentioned in paragraph
(E) The Alexandrine synthesis attempted to discourage 2, and a terminated marriage contract is not
clandestine marriages, but there is no mention of clandestine.
whether this approach was due to parental concerns
about such unions. 5. (E) Global (Purpose of Passage)
12
Section 2: Reading Comprehension
(B) While it is plausible that certain modern marriage marriage contracts that are legally valid and those that
laws can be traced back to Alexandrine doctrine, the comply with ecclesiastical requirements. Answer choice
author makes no connection between Alexandrine (C) best captures this idea.
doctrine and modern marriage laws. (A) is a 180. The paragraph elaborates on a distinction
(D) is a train wreck. It is almost as if the LSAT writers mentioned in the first paragraph; there are no divergent
pulled words from the passage at random to create interpretations.
this answer choice. The author never suggests other (B) The rule mentioned in the first paragraph was that
studies of medieval marriage have “misread” any couples could get married without obtaining the
aspect of the rituals, and never describes the legal and consent of their parents. The second paragraph
ecclesiastical rituals as being particularly complicated contains no exception to this rule.
or susceptible to misunderstanding. The author does
(D) There is neither a summar y of traditional
not describe “rituals” for public announcement or ritual
interpretations of a topic nor is there an introduction of
solemnization of marriage.
a new interpretation in paragraph 2.
(E) offers up an irrelevant comparison. The contrast
(E) The first paragraph did not contain an argument, so
between Donahue and other scholars has nothing to do
the second paragraph could not possibly set forth
with a shift in focus from legal to ecclesiastical views.
objections to a prior argument.
7. (D) Detail
13
PrepTest A Explained
Passage 2: Nontraditional Black Women (A) The first sentence of the passage states that
Filmmakers nontraditional black women filmmakers share with
radical filmmakers a problematic relation to
The passage announces its Topic (nontraditional black mainstream cinematic practice (as opposed to
women filmmakers) and Scope (their problematic conservative techniques of editing and lighting, which
relation to mainstream, realist filmmaking) in its first are but two aspects of mainstream practice). The
sentence. However, the first paragraph then seems to remainder of the passage, however, does not explore
abandon those nontraditional black women filmmakers that common bond between the two groups of
in order to discuss the differences between realistic filmmakers.
cinema and avant-garde filmmaking. Realistic films (C) According to the passage, some experimental
create the illusion that cinema is like life. Avant-garde filmmakers use various techniques to highlight the
or radical films use techniques that highlight the artificiality of the cinema, but the nontraditional black
artificiality of the filmmaking process. women filmmakers described in the passage made
The second paragraph returns to nontraditional black films that were experimental not because of their form
women filmmakers, to note that some of them used but because of their content.
realistic cinematic techniques (the documentar y (D) The author makes no attempt to describe the
format) to tell nontraditional stories. The Purpose and impact black women filmmakers have had on cinematic
Main Idea are now clear: the author argues that practice. Rather, the author describes how some black
although some might say that realistic documentaries women filmmakers have used the techniques of
cannot be considered “experimental,” the realistic cinema to explore nontraditional subject
nontraditional subject matter “rightfully” allows these matter.
works to be classified as experimental. (E) Information about the mainstream media is only
The third paragraph provides two examples of films provided to illuminate the discussion of nontraditional
made by nontraditional black women filmmakers. One black women’s films. The author never says or implies
example represents films that presented stories of that mainstream filmmakers prefer to avoid
ordinary women whose experiences have usually been controversial topics in their films.
trivialized or ignored by mainstream media. The other
example represents films that provide accounts of 10. (D) Inference
black public figures, who may not have received public
recognition. Watch out for half-right, half-wrong answer choices on
Inference questions.
Our Roadmap might look something like this:
Going back to the line reference mentioned in the
¶1: realist versus radical filmmakers question, we see that the “prevailing notion” was that
documentaries using realistic cinematic techniques
¶2: some nontraditional black women filmmakers: realist could not be considered “experimental.” The author
techniques and nontraditional subject matter then notes that the content of a realistic work could
¶3: examples of black women filmmakers’ documentaries “rightfully” qualify the work as experimental. That’s
what answer choice (D) says.
9. (B) Global (Main Idea) The remaining answer choices all start out with
promising language, indicating the author’s
The Main Idea will never be encompassed by a disagreement with the “prevailing notion” referred to in
description of details set forth in the passage. the question stem. However, the second half of each
The first paragraph, while primarily concerned with answer choice strays from the author’s point.
contrasting the varying techniques used by mainstream Nothing in the passage suggests that mainstream
realist filmmakers versus radical filmmakers, sets the filmmakers advocated a narrow definition of “technical
stage for a discussion of how nontraditional black experimentation,” (A). The passage never identifies
women filmmakers used realistic cinematic techniques the proponents of the prevailing notion.
(in documentaries) to make films that were The author never suggests that conservative cinematic
“experimental” because of their subject matter. (B) techniques possess a “unique” potential for
captures that idea. expressing radical ideas, (B). Rather, the black
women’s films were experimental because even though
14
Section 2: Reading Comprehension
they used conservative techniques, the subject matter wish to admit or know. That’s answer choice (C) in a
was nontraditional. nutshell.
(C) contradicts the author’s point that nontraditional (A) The passage never suggests that the theorists
black women filmmakers were using realistic, consider mainstream filmmakers’ audiences to be
mainstream techniques in their films. uneducated or otherwise unable to appreciate films
(E) The author never contends that form is not as that employ experimental techniques.
important as content. All the author says is that (B) If anything, the passage implies that nontraditional
content may be just as good a measure as form is for black women filmmakers seek to combat majority
determining whether a film is experimental. culture ignorance about minority cultures. The author
does not credit the theorists mentioned in the passage
11. (D) Detail (EXCEPT) with an opinion on this subject.
(D) This answer choice is wrong on several counts. The
A question that asks you to identify the one detail not
author never attributes to the theorists the opinion that
mentioned in the passage may be a good one to leave
mainstream filmmakers avoid the documentary form.
until later.
To the contrar y, the author suggests that the
With a question like this, all we can do is go through documentary form is mainstream, conservative, and
the answer choices one at a time and see if they are realistic. In the beginning of the passage, the author
specifically mentioned in the passage. specifically states that realistic techniques (e.g., those
(A) In lines 52–54, the author discusses how the used in documentaries) create illusions of visual
documentaries made by black women filmmakers call continuity.
the thematic assumptions of realistic filmmaking into (E) The passage never suggests that the theorists have
question. a theory on why mainstream filmmakers don’t make
(B) In lines 54–63, the author notes that the more experimental films, and there is no suggestion by
documentaries may serve to examine the lives of black the theorists, the author, or anybody else in the
public figures who have not received substantial public passage, that the mainstream filmmakers lack the
recognition. technical knowledge to create experimental films.
(C) In lines 49–52, the author talks about the way that
13. (D) Global (Purpose of Passage)
the documentaries present the stories of women
whose experiences have been ignored by the Use a vertical scan of verbs to eliminate wrong
mainstream media. answer choices on a question that asks you to
(D) Nowhere in the passage does the author suggest identify the Purpose of the passage.
that the documentaries influence how women are
The opening sentence of the passage identifies the
portrayed in mainstream media. This is our correct
“problematic” relation between nontraditional black
answer choice.
women filmmakers and mainstream, realist cinematic
(E) Lines 63–64 contain the author’s observation that practice. The remainder of the passage describes
the documentary films about relatively unknown black (examines) that problematic relation, choice (D).
public figures serve to urge audiences to think about
(A) While two specific films are mentioned in the last
who gains public recognition and why.
paragraph, the discussion of the films provides
evidence about the relation between the nontraditional
12. (C) Inference
black women filmmakers and realist cinema practice.
Pay close attention to multiple viewpoints in a The films are contrasted (one film being about ordinary
passage. women and one film being about black public figures),
but there is no particular assessment of the films.
Now we are concerned with the views of “theorists”
(B) The author provides a brief critique of realist
referred to in line 27, as opposed to the views of the
techniques by “some theorists,” but does not describe,
author, with respect to mainstream filmmakers.
let alone critique, the subject matter of realist films. To
Looking back to the reference, we see that the
the extent that there is any critique of realist cinema, it
theorists believed that mainstream, realistic film
is part of the evidence the author uses to explore the
techniques create an illusory “reality” for filmgoers,
“problematic relation.”
shielding them from a reality that audiences do not
15
PrepTest A Explained
16
Section 2: Reading Comprehension
¶4: Seafloor spreading supports theory This question asks us to find the answer choice
describing a situation most parallel to Jeffreys’s
approach to Wegener’s theory. Looking back to the
17
PrepTest A Explained
author’s descriptions of Jeffreys’s approach in reason why basalts retain their magnetic field
paragraph 2 and at the end of paragraph 3, we recall alignment. The fact that basalts and seafloor spreading
that Jeffreys’s approach was “casual,” “brusque,” and are not discussed in the same paragraph is another
not thoughtful or well-reasoned. We’ll need to test each clue that this is not the correct answer.
answer choice to find a similar approach: (C) The author notes that the earth’s poles have
(A) The botanist draws a conclusion about two species wandered over the past few hundred million years, but
based on “super ficial” examination of their the reasons why remain a mystery to the reader of this
appearance. This one looks good, but we’ll check the passage.
others to be sure we haven’t missed anything. (D) The passage says nothing about the composition of
(B) According to the passage, Jeffreys was not the earth’s giant crustal plates.
wandering aimlessly, like a driver in an unfamiliar city,
trying to find a street without a map. Rather, Jeffreys 17. (C) Inference (Author’s Attitude)
knew the city (Earth), but was unwilling to consider that
the city might not have always been configured the way When more than one viewpoint is discussed in a
it appeared to him. passage, watch for clues about the author’s attitude
toward the proponents of each view.
(C) Jeffreys’s problem wasn’t that he chose an
inappropriate or disfavored field of study within his This passage is scattered with evidence of the author’s
discipline. lack of respect for Jeffreys’s “scholarship” on which
(D) Jeffreys did not base his opinions on the Jeffreys based his rejection of Wegener’s theory. In
consensus of either the public or those within his field. paragraph 2, the author notes that Jeffreys dismissed
He simply dismissed Wegener’s theory without giving Wegener’s theor y “[a]pparently after casually
adequate consideration to the evidence that Wegener observing the shorelines on a globe….” Paragraph 3
might be correct. describes the thoughtful research that supported the
theory of continental drift, done by researchers who
(E) For a psychiatrist to base his treatment decisions
were “[d]isturbed by Jeffreys’s obviously perfunctory
on his patients’ past histories might actually be sound
observation.” After describing this new evidence that
medical practice. However, Jeffreys based his opinions
the continents had moved with respect to one another,
on superficial and unreasoned reactions to the theory
the author states, “True to form, Jeffreys brusquely
and to the evidence supporting Wegener’s theory.
rejected” the studies. The author then relates how
(A) is the only answer choice that captures the lack of Jeffreys’s “casual disdain” for the observational data
intellectual rigor in Jeffreys’s approach, and it is our that supported Wegener’s theor y led some field
correct answer choice. geologists so suggest that Jeffreys’s classic book, The
Earth, should be retitled, An Earth. (Those geologists
16. (E) Detail can really come up with cutting insults.) (C) captures
this sense of Jeffrey’s arrogant and feckless dismissal
Never rely on your memory to answer a Detail
of Wegener’s theory.
question.
(A) According to the author, Jeffreys did not ignore
The keywords “seafloor spreading” give us an easy Wegener’s theory. We are told that Jeffreys considered
hook to scan for, and we find information about Wegener’s theory and dismissed it without any real
seafloor spreading in paragraph 4. Lines 50–54 tell us consideration. Moreover, the context of the reference
that evidence of seafloor spreading explained the shows that the author was not using these words to
puzzle of why the ocean basins are so much younger describe Jeffreys.
than the continents. Answer choice (E) speaks to that
(B) While the author might agree that Jeffreys’s
very point.
response to Wegener’s theory was “very poor,” the
(A) The author indicates that seafloor spreading occurs context of that quote shows that the phrase referred
as the earth’s giant crustal plates move apart, but not to Jeffreys but to the apparent fit between the
seafloor spreading is not posited as the “reason for shorelines of Africa and South America.
the existence” of those giant crustal plates.
(D) The author describes S.W. Carey and not Jeffreys,
(B) The author states that basalts retain a magnetic as using “careful techniques.” The careful reader could
field in alignment with the earth’s magnetic field at the have eliminated this answer choice immediately upon
time the basalts were formed, but never suggests any noting how positive the phrase is—this author clearly
18
Section 2: Reading Comprehension
considers Jeffreys’s research techniques to be Carey did not decline to investigate Jeffreys’s critique.
slipshod and lackadaisical, not “careful.” To the contrary, he applied “careful techniques” to the
(E) Jeffreys’s opinion never became the consensus problem, and found evidence contrary to Jeffreys’s
view. Rather, the observational data that Jeffreys opinions.
scorned, along with the evidence of seafloor spreading,
turned Wegener’s theory (along with plate tectonics) 20. (A) Logic (Weaken)
into the consensus view.
One way to weaken an argument is to falsify some of
the evidence used to support the conclusion.
18. (C) Logic (Function)
The author cites three key pieces of evidence to
Context is the key to understanding the function of support Wegener’s theory of continental drift: the
anything mentioned in the passage. match between the continental margins of Africa and
Mention of the field geologists’ insulting renaming of South America, the varying alignment of magnetic
Jeffreys’s classic book comes at the end of a fields in basalts of the same age in North America and
paragraph describing careful research by scholars who Europe, and the evidence of seafloor spreading due to
were “disturbed” by Jeffreys’s approach. The insult new rock being formed at the midocean ridges, which
shows that not only was the author unimpressed by explains how the ocean basins can be younger than the
Jeffreys’s brusque dismissal of the evidence continents (paragraphs 3 and 4). Answer choice (A)
suppor ting Wegener’s theor y, but also Jeffreys’s undermines this evidence by offering the possibility
colleagues thought little of Jeffreys’s scholarship in that the ocean basins are older than the continents.
this regard. That’s answer choice (C). (B) The possibility that new techniques could allow for
(A) The author describes only one of Jeffreys’s ideas: more accurate mapping could go either way with
his belief that Wegener was wrong. respect to the theory of continental drift. Better
mapping would not necessarily render the theory less
(B) The insult by the field geologists is itself criticism
likely to be valid.
of Jeffreys’s work. The author justifies criticism of
Jeffreys’s work due to its lack of substance. (C) According to the passage, the planet’s magnetic
poles have wandered over the course of a “few
(D) The insult explains nothing. It is simply evidence
hundred million years” (line 32). Evidence that basalts
that Jeffreys’s colleagues found his objections to
formed (and magnetized) in North America and Europe
Wegener’s theory to be insubstantial.
during the past 100 years are magnetically aligned in
(E) Other than identifying the field geologists as critics the same direction says nothing about whether the
of Jeffreys, the author makes (and thus supports) no continents on which those rocks are found have moved
assertions about those critics. relative to one another over the eons.
(D) is completely consistent with the theor y of
19. (D) Inference
continental drift, since the author tells us that basalts
When a passage contains multiple viewpoints, stop become magnetically aligned at the time they are
and be sure that you are clear on which of those formed. This is a restatement of one of the pieces of
viewpoints are at issue for the question at hand. evidence supporting the theory, not a weakener.
(E) This answer choice is too broad and vague to
Paragraph 3 begins with the author describing Carey as
constitute a weakener of the argument. Were the
“disturbed” by Jeffreys’s “obviously per functor y”
geological phenomena that Jeffreys “carefully” studied
observations. Carey not only discounted Jeffreys’s
at all related to the evidence supporting continental
opinions because of Jeffreys’s inadequate research,
drift? If not, then those studies could not possibly
but also performed his own “careful” research to
undermine the theory of continental drift. Even if
counter Jeffreys’s critique. (D) best matches that
Jeffreys had made “careful observational studies” of
evaluation of Carey’s opinion about Jeffreys’s work.
geologic phenomena relevant to the theor y of
We can quickly eliminate (A), (B), and (C) because all continental drift, were those studies made before or
of them imply that Carey agreed with Jeffreys’s after he criticized Wegener’s theory? If they were
conclusions. completed afterward, for all we know, Jeffreys changed
(E) captures the idea that Carey found Jeffreys’s his mind and became a continental drift supporter.
appraisal of Wegener’s theory to be deficient, but
19
PrepTest A Explained
Passage 4: Maravall and the European 21. (B) Global (Main Idea)
Baroque Watch out for answer choices that employ a faulty
use of details.
The section ends with a Social Sciences passage
detailing Maravall’s theory that the European baroque Our Roadmap tells us that this passage is all about
period was not just an aesthetic style, but also a Maravall’s view of baroque culture as a tool used by
discrete historical period with political, social, and the ruling classes to manipulate and control society at
cultural phenomena that foreshadowed cer tain large, and that the author believes that Maravall
aspects of the modern world. overstates the success of this strategy for social
Paragraph 1 announces the Topic: the European control. That’s answer choice (B).
baroque. The Scope is Maravall’s theory about the (A) This answer choice is tempting, as it talks about
baroque being more than an aesthetic style. The the concept of the baroque being more than an
author’s Purpose and Main Idea are not completely aesthetic style, and notes Maravall’s concept of
clear until the last paragraph, where the author, after baroque culture as a way that rulers manipulated and
describing Maravall’s interpretation, offers mild controlled the general public. However, the passage
criticism of Maravall’s theory. never indicates that Maravall’s interpretation of the
In paragraph 2, the author discusses Maravall’s European baroque is a “recent” development in
argument that the baroque was a period when the historical analysis.
ruling classes asserted control over a society that had (C) This is another tempting wrong answer choice,
become unsettled by the Renaissance’s “liberating because the passage does tie the European baroque
forces of criticism and opposition.” to the Renaissance, but not as an “expansion” of the
Paragraph 3 is where the author explains Maravall’s social and intellectual developments of the
view of how the monarchy and aristocracy controlled Renaissance. Rather, the baroque is seen as a
the masses, using “grandiose ar tifice” such as reaction to the unsettling effect of the Renaissance on
fireworks displays, theater, and religious festivals as society at large. Further, the author suggests that
tools of manipulation. Maravall was overly influenced by his experience of
living under the Spanish dictator Franco, as opposed to
In paragraph 4, however, the author argues that any suggestion that Maravall’s ideas were colored by
Maravall’s interpretation goes too far, and that he his “focus” on Spain.
tends to exaggerate the capacity of rulers and
aristocrats to manipulate society for their own (D) starts off in a promising way, talking about
ideological ends. Moreover, according to the author, Maravall’s theory of the ruling class using culture as a
the theatrics and symbolism of the baroque negatively means of social control, but the term “refuted” is much
affected members of the ruling class, “dangerously too strong for the mild criticism offered by the author
isolating” them from the outside world and leading the in the final paragraph.
monarchy and aristocracy into a credibility gap that (E) This answer choice is good as far as it goes, but it
made them less effective in controlling their societies. leaves out the whole last paragraph, where the author
Our Roadmap might look like this: suggests that Maravall might have been overly
influenced by his desire to see a foreshadowing of
¶1: Maravall—baroque as political, social, cultural modern society in the baroque era, and by his
phenomenon, pre-modern experience of living under the dictatorship of Franco in
Spain. This answer choice also fails to capture the
¶2: Baroque culture: control and containment author’s point that the strategy of using baroque
culture to control a society was not necessarily
¶3: Political controls: repressive but enticing effective, and may even have been counterproductive
¶4: Author: M. overstates rulers’ powers to control at times. This failure to account for a significant
society portion of the passage makes this an incorrect answer
choice for a Global question.
20
Section 2: Reading Comprehension
22. (C) Inference (Author’s Attitude) (C) While a “directive” may be a codification of rules or
instructions, Maravall’s view of baroque culture was
Your Roadmap should clue you in to places within the that it served to entice the interest of the masses, and
passage where the author’s attitude is apparent. redirect their desire for novelty into areas where that
Our Roadmap indicates that the author’s criticism of desire would pose no threat to the political order. There
Maravall’s theory comes in paragraph 4, beginning with is no sense that baroque culture was somehow bound
line 44. Since we are looking for an answer choice that by some type of code.
reflects the author’s attitude “in context,” we can (E) The author uses Maravall’s concept of a “directive”
quickly eliminate answer choices (A) and (B), which culture as the springboard for a discussion of how the
point to earlier sections of the passage. baroque culture served to entice the general public into
(C) Lines 47–48 and the phrase “tends to exaggerate” submitting to the authority of the ruling classes.
precisely describe the author’s attitude toward “Compelling” is too strong of a term to describe this
Maravall. That’s our right answer. enticement.
(D) The phrase “own ideological ends” at line 49 is in
24. (D) Inference
the right paragraph, but in context, refers to the
motives of rulers in promoting the baroque culture, not Your understanding of the scope of the passage can
the author’s attitude toward Maravall’s concept of help you eliminate wrong answer choices on an
baroque culture. Inference question.
(E) Likewise, the phrase “wholly counterproductive,” in
The author tells us that Maravall believed that during
context, refers to the attempts by the ruling class to
the baroque era, monarchs used the enticements of
use baroque culture as a means of social control. Not
baroque culture to defend traditional order and values
a good answer choice when placed into context.
that had been disrupted during the tumult of the
Renaissance. That’s answer choice (D).
23. (D) Inference
(A) The passage never suggests that Maravall
It is especially important to check for context when regarded the monarchs as increasingly indifferent to
the question involves the meaning of a word used in unfavorable public opinion. To the contrary, Maravall
the passage. argued that every aspect of baroque culture was
related to the importance of manipulating public
We see the word “directive” used most frequently as a
opinion.
noun meaning “order,” but in this passage the author
uses the word as an adjective, describing Maravall’s (B) According to the passage, the aristocracy was
concept of the baroque as a culture of control and aligned with the monarchs, part of the ruling classes
containment, designed to “reintegrate and unite” a who used baroque culture to achieve ideological ends.
society that had been disrupted. This sense of baroque (C) The author, and not Maravall, suggests that the
culture as being a tool of manipulation gives the use of monarchs were themselves captivated by baroque
the word “directive” a meaning along the lines of culture, which caused the ruling class to become
“guiding,” answer choice (D). “dangerously isolated” from the outside world.
(A) “Direct” may mean “straightfor ward,” but (E) The passage never suggests that Maravall saw the
Maravall’s view of the Baroque as a “directive” culture monarchs of Spain and England (or any other
reflected a sense of manipulation of the public. Not at monarchs) as striving for cultural preeminence among
all “straightforward.” the countries of Europe.
(B) Maravall viewed the “directive” Baroque culture as
a way of changing society (for the benefit of the ruling 25. (C) Inference
classes), but the word “evolving” does not capture
Research, then pre-phrase your answer choice.
Maravall’s point. “Evolving” suggests a process that is
not necessarily directed by outside forces, whereas Baroque theater is mentioned in paragraph 3, as one
Maravall viewed the changes in society during the of the means by which the ruling class delighted and
Baroque era as the result of the conscious effort of the redirected the general populace, so as to avoid
ruling classes to reestablish their authority and control challenge to the political order. Answer choice (C) says
over society. just that.
21
PrepTest A Explained
(A) While Maravall saw baroque culture as a response distract the populace from the repressive nature of the
to the European social and economic crisis after the monarchy. If many baroque era works of art in fact
Renaissance, the passage never suggests that expressed opposition to the monarchy, as in (D), that
Maravall saw baroque theater as a way to spur would undercut Maravall’s theor y about baroque
economic growth. culture.
(B) In Maravall’s view, baroque theater, along with (A) The author notes that Maravall lived under the
other diverting spectacles, was intended to channel Franco dictatorship, and suggests that Maravall’s life
and manipulate public opinion, not reflect any experiences colored his view of baroque art. The
consensus of public opinion. precise nature of Maravall’s life experiences, however,
(D) The emerging principle of individual liberty was says nothing about whether or not baroque art and
precisely what baroque culture was intended to culture was a way of controlling the population for the
suppress. benefit of the ruling classes.
(E) Baroque theater was intended to entice, not terrify, (B) Both the nobility and the monarchs are described
the citizenry. It was intended to deflect their attention by Maravall as being part of the “ruling class” that
away from the repressive nature of the monarchy. used baroque art to manipulate public opinion. Any
purpor ted difference in the number of ar tworks
26. (C) Global (Purpose of the Passage) commissioned by the nobility versus the monarchs
would not affect the validity of the argument that both
Be ruthless in eliminating answer choices that do not groups used art to achieve their ideological ends.
match your Roadmap. (C) If we were to learn that baroque art provided an
Our Roadmap reminds us that the first three idealized depiction of the monarchy and aristocracy,
paragraphs of the passage are spent explaining that would fit right in with Maravall’s view that baroque
Maravall’s theory of baroque culture, and the last art was used to strengthen the position of nobility and
paragraph briefly sets forth the author’s mild criticism royalty within that society.
of Maravall’s theory. Answer choice (C) follows that (E) Evidence that the dictator Franco sought to control
Roadmap quite nicely. Spanish society by cultural means would, if anything,
(A) While the author does not entirely agree with strengthen Maravall’s argument, as another example
Maravall’s theor y, she does not set for th any of those in power using art and culture to manipulate
competing theory to contrast with Maravall’s theory. and control society at large.
(B) The author challenges Maravall’s viewpoint, but
offers no indication as to whether the view of Maravall
and “some” other scholars about the European
baroque is widespread.
(D) Given the tentative and mild nature of the author’s
critique of Maravall’s theory, that critique cannot fairly
be called an “opposing” argument. Further, the author
makes no attempt to reconcile that critique with
Maravall’s views.
(E) “Explain” is a good verb for beginning the answer
choice, but it’s all downhill from there. The author
never describes any “consequences” following from
Maravall’s view of the baroque as a political construct.
22
Section 3: Logic Games
23
PrepTest A Explained
24
Section 3: Logic Games
25
PrepTest A Explained
(A) Pick f to present on Monday and g may or may not Salesperson g and product O must be scheduled for
present product O. the same day, even though g presents product P. Either
(B) Schedule i for Tuesday, and j could go to either f or h must present with g, because we need to keep
Monday or Wednesday. them separated. We still have the option of using either
i or j to present product N on Tuesday. With N, O, and
(C) If j presents N on Tuesday, g remains able to
P placed on Tuesday and Wednesday, products L and
present on Monday or Wednesday.
M must be presented on Monday. Our new drawing
(D) If f and j are scheduled for Monday, h, g, and O reflects these new deductions:
must be scheduled for Wednesday, but O could be
Mon. Tues. Wed.
matched with either presenter.
Pres. ___
f/h ___
i/j ___
i/j ___
f/h ___
g
(E) is the only answer choice where the “if” clause
forces the proposed result to occur. If h and i present Prod. L/M
___ L/M
___ ___
N ___
O ___ P
products on Wednesday, then product O (along with g
and f) must be assigned to Monday. Working through the answer choices, we see
Mon. Tues. Wed. (A) Salesperson h could present product O.
Pres. ___
f ___
g ___
j ___
h ___ i (B) Salesperson i could present product N.
Prod. ___
O/? ___
O/? ___
N ___ ___ (C) For i to present product O, i would have to be
placed on Wednesday. But if we did that, f and h would
be forced to go on the same day, and that would violate
7. (D) “Must be true” / “if” clause our first rule, so this answer choice must be false. On
Test Day, we would stop right here and move on to the
Knowing where the game provides many options is as next question. For the record:
important as knowing where the options are limited.
(D) Our sketch proves that f could make a presentation
If h and i are assigned to Monday, f and g (along with on Monday.
product O) must go to Wednesday. Sharp-eyed (E) Salesperson f could just as easily make a
students will note that the “if” for this question is the presentation on Wednesday.
mirror image of the “if” in the correct answer choice for
the previous question. Just reverse Monday and 9. (D) “If” clause / “Could be true”
Wednesday on the previous drawing and you have your
answer, (D). Once you have laid out the possibilities, quickly test
Mon. Tues. Wed. answer choices against those possibilities.
Pres. ___
h ___i ___
j ___
f ___g This question places products M and P on Monday,
Prod. ___ ___ ___
N ___
O/? ___
O/? which forces products O and L to Wednesday.
Salesperson g goes with O to Wednesday, along with
either f or h. Monday’s presenters are f or h, and i or j.
(A), (B), and (E) can be quickly eliminated from Drawing out those possibilities gives us:
consideration, because in this game, products L, M,
Mon. Tues. Wed.
and P are floaters and could be placed anywhere
except Tuesday. Pres. ___
f/h ___
i/j ___
i/j ___
f/h ___g
(C) The rules place product N on Tuesday, to be Prod. ___
M/P ___
M/P ___
N ___
L/O ___
L/O
presented by either i or j. If i is scheduled for Monday,
j must present product N on Tuesday. We can now use our sketch to locate the answer choice
that could be true, and eliminate answer choices that
8. (C) “If” clause / “Could be true EXCEPT” must be false.
When the wrong answer choices could be true, the (A) Salesperson g must present on Wednesday and
correct answer choice must be false. product P is presented on Monday, so this answer
choice must be false.
The new rule for this question requires g to present
(B) Product O must be presented on Wednesday, and
product P on Wednesday. The first thing to do is to
i must make a presentation either Monday or Tuesday,
repeat Steps 3 and 4 of the Kaplan Method.
so this answer choice must be false.
26
Section 3: Logic Games
(C) Product L must be presented on Wednesday, and j that j presents neither product O nor product P, it must
must make a presentation either Monday or Tuesday, be true and is the correct answer.
so this answer choice must be false.
(D) Salesperson f is free to make a presentation on
Monday, so this is our correct answer choice. On Test
Day, we would choose it and move on. Just to round
things out for our practice:
(E) Salesperson g must make a presentation on
Wednesday, so this answer choice must be false.
27
PrepTest A Explained
Game 3: Academic Office Assignments ____ ____ ____ ____ ____ ____ ____
101 102 103 104 105 106 107
At first glance, this appears to be a fairly No F No F
straightforward Sequencing game. The only oddity is
that the entities are divided into subgroups. The LSAT No G No G
writers never create subgroups without a reason, so we Rule 3 requires G and r to be separated. We can note
want to be on the lookout for ways that the subgroups this rule:
will come into play. We can visually distinguish between Never G r or r G
the subgroups by using upper case, lower case, and Rule 4 is a tricky “unless” rule. Translating it into an
cursive letters for our entities in each subgroup. if/then statement, the rule states that if V and W
Situation: Academic offices occupy consecutive offices, then r must be next door to
Entities: Professors: F and G; Lecturers: q, r, and s; V. Our scratchwork for this rule should look something
Instructors: V and W, and their offices, rooms 101 like this:
through 107 V W or W V Æ r V W or W V r
Action: To sequence the assignment of offices. Lecturers q and s are floaters.
Limitations: Each person is assigned one of seven Key Deductions: This game is wide open. We have
offices. The offices are lined up along one side of a rules that prevent certain entities from being placed
hallway, and are numbered consecutively. next to one another, but the accumulation of rules
The Initial Setup: Most of the time, when entities are never reaches the point of forcing any particular
divided into subgroups, the best practice is to include combination of entities. There is no Bloc of entities
space in the initial sketch to reflect information about with limited placement possibilities. No rules break the
the subgroups. In this particular game, however, there possibilities into Limited Options. There are no
are very few entities in each subgroup, and so it seems Established Entities, and with a straight sequencing
simpler to translate the two rules about the subgroups game, there is no Numbers issue to consider. There is
into rules about the relationships between the Duplication of entities within the rules, in that G, r, V,
individual entities. That way the game remains one of and W show up in multiple rules, but the duplication
straight sequencing. We can still keep track of our serves only to narrow the possibilities without leading
subgroups when necessar y, using the visual to any deductions.
differences we have established in the way we write It is unusual for the LSAT writers to create a game with
the entities for each subgroup. no obvious way of combining any of the rules, but once
we have analyzed and drawn the rules and gone
Profs: F G Lect: q r s Instr: V W through the Kaplan BLEND process, we can move on to
____ ____ ____ ____ ____ ____ ____ the questions with ever y expectation that our
understanding of the rules will enable us to answer the
101 102 103 104 105 106 107 questions.
28
Section 3: Logic Games
11. (B) “If” clause / Partial Acceptability answer choice (B). Now we can start testing the
remaining answer choices to see which one must be
Attack Partial Acceptability questions the same way placed next to r.
you attach ordinary Acceptability questions: use the
(A) If we try to place F in a spot other than rooms 103
rules to eliminate wrong answer choices.
or 105, the only possibilities would be 102 or 106
This question asks us to consider which entities could (101 and 107 are prohibited under Rule 2, and 104 is
be assigned an office in between F and G. The correct already assigned to r). Because G cannot be placed
answer choice will be the one in which either of the two next to r under any circumstance and like F, is
entities listed could fill that spot. We use the rules to prohibited from occupying 101 or 107, G would also
quickly eliminate the wrong answer choices. have to be placed into room 102 or 106. But with F and
Rule 1 tells us that neither V nor W can be in an office G taking offices in 102 and 106, there would be no
next to F or G. That eliminates answer choices (C), (D), room V or W could be assigned without violating Rule
and (E). 1. So as it turns out, (A) is our good answer choice.
Rule 3 says that G and r never occupy adjacent offices. We can prove that neither q (C), V (D), nor W (E) must
That eliminates (A), and leaves (B) as the correct be placed in 103 or 105 by the following hypothetical:
answer.
____
V/W ____
q/s ____
V/W ____
r ____
F ____
G ____
q/s
12. (C) Partial Acceptability 101 102 103 104 105 106 107
Practice translating “unless” rules until the
14. (A) Maximum number of spaces between two
translation process becomes second nature for you.
entities
This question demands a complete understanding of
the implications of Rule 4, the “unless” rule in the When asked to identify a maximum for a game, start
game. Properly translated, that rule states that if V and with the biggest answer choice and work your way
W are assigned adjacent offices, then V and r must down.
also be assigned adjacent offices. We test each (E) For there to be five rooms between F and G’s offices,
answer choice against the rule, to find the one that F and G would have to be assigned rooms 101 and 107,
complies with the Rule 4. a violation of Rule 2. Eliminate this answer choice.
(A) V and W are adjacent, but r is placed next to W. No (D) For there to be four rooms between F and G’s offices,
good. either F or G would have to be placed in room 101 or
(B) V and W are adjacent, but r is not adjacent to V. 107, a violation of Rule 2. Eliminate this answer choice.
Another rule-breaker. Eliminate it. (C) For there to be three rooms between F and G’s
(C) V and W are not adjacent. The rule does not come offices, F and G would have to be placed in rooms 102
into play. Hold on to this answer choice. and 106. If we did that, we could safely place V or W in
(D) V and W are back to being neighbors, but r is not room 104, but we would have no place to assign the
adjacent to V. Eliminate. second instructor without violating Rule 1. Another
wrong answer choice.
(E) V and W are adjacent, but r is placed next to W. Not
good enough to satisfy Rule 4. (B) Likewise, if there are two rooms between F and G’s
offices, we could only give one Instructor a room
(C) is the only answer choice not eliminated by Rule 4,
without violating Rule 1. Eliminate this one.
so it is the right answer.
(A) We have eliminated all of the other answer choices,
13. (A) “If” clause / “Must be true” and so (A) must be the right answer. Even the most
compulsive student would not need to map out this one
If the right answer “must be true,” test answer to confirm that it’s possible, because Question 11
choices by trying to create a hypothetical where the already told us to assume that F and G were assigned
answer choice is not true. rooms that have exactly one room between them. The
LSAT writers never ask us to assume something that
Our new rule (the “if” clause) places r in room 104. We
would violate the rules, so we know that F and G can
are looking for the entity that must be placed next to r,
indeed be spaced one room apart.
in room 103 or room 105. Rule 3 prohibits G from
being placed next to r, so we can quickly eliminate
29
PrepTest A Explained
15. (E) “CANNOT be true” rooms 101 or 107. If we try to put the professors into
the even-numbered rooms, the rules would force V and
When asked to identify an entity that cannot occupy W to be next to one another, in violation of our new “if.”
a certain spot, first consider the entity among the So the only places we can put the professors are in
answer choices that is the most limited by the rules. rooms 103 and 105. Let’s draw out our two options:
Of our five answer choices, G and V are the entities Option I:
most bound by the rules, so we’ll try them first.
____
W/V ____
q/s ____
G ____
q/s ____
F ____
r ____
W/V
(B) G has no problem being assigned to room 104, as
this hypothetical proves: 101 102 103 104 105 106 107
Option II:
____
W ____
r ____
F ____
G ____
q/s ____
q/s ____
V
101 102 103 104 105 106 107 ____
W/V ____
r ____
F ____
q/s ____
G ____
q/s ____
W/V
101 102 103 104 105 106 107
Eliminate answer choice (B).
(E) V, however, is another story. If we try to put V into A quick comparison of our answer choices to these two
room 104, Rules 1 and 2 prohibit F and G from options proves that (E) is the only one that must be true.
occupying any rooms but 102 and 106. We already
have proven that they cannot occupy those rooms, and 17. (D) “If” clause / “CANNOT be true”
so (E) is our correct answer choice. For the record,
Turn negatives into positives whenever possible.
(A) F could be placed into room 104, as shown by this
hypothetical: Our new “if” clause prohibits F and G from being
assigned rooms that are next to each other. We know
____
W ____
q/s ____
G ____
F ____
r ____
q/s ____
V from question 14 that if F and G are not next to one
another, they are separated by exactly one room. As it
101 102 103 104 105 106 107
turns out, they were separated by one room in
Question 16, where we proved that either V (B) or W
Answer choices (C) q and (D) s could be eliminated
(A) could be placed into room 107. Eliminate those
simply on the grounds that with both of them being
answer choices.
floaters, they each have exactly the same likelihood of
being the correct answer choice. Since there is no As floaters, s (C) and q (E) have exactly the same
basis for distinguishing between the two, and only one likelihood of being a good answer choice, which means
answer choice can be correct, they both must be they can both be eliminated. Not comfortable with
incorrect answer choices. Alternately, we can eliminate summarily eliminating the two floaters? Tr y this
them with the following hypothetical: hypothetical:
____
W ____
V ____
r ____
q/s ____
F ____
G ____
q/s ____
W ____
V ____
r ____
F ____
q/s ____
G ____
q/s
101 102 103 104 105 106 107 101 102 103 104 105 106 107
30
Section 3: Logic Games
Game 4: Rug Exhibition always select at least one oval wool rug (as an
Established Entity):
This game is a gift from the LSAT writers—a straight
game of Selection, with five of the six questions boiling
Select 4 to 6:
down to issues of Acceptability.
OW OW RW RW RW OS RS RS
Situation: Rug Exhibition
Entities: The rugs: two oval wool (OW) rugs, three
rectangular wool (RW) rugs, one oval silk (OS) rug, and Because one of the two to three wool rugs that we
two rectangular silk (RS) rugs select must be oval, that means that at most, two
Action: Selection rectangular wool rugs are chosen. Cross off one
Limitations: Four to six of the eight rugs will be selected. rectangular wool rug:
Initial Setup:
Select 4 to 6:
For a Selection game, we create a roster of entities,
OW OW RW RW RW OS RS RS
circling entities that get selected, and crossing off
entities that are rejected. When there are a number of
entities of the same type, repeat the entities the
Because we have several rules about numbers (four to
appropriate number of times in your roster:
six rugs must be selected, two or three oval rugs must
Select 4 to 6: be selected, and two or three wool rugs must be
selected), we should consider the numerical
OW OW RW RW RW OS RS RS possibilities for this game.
If two wool rugs are chosen, then we must choose two
The Rules: to three silk rugs. If only two wool rugs are chosen, we
Rule 1 requires us to choose at least two out of the will never have six rugs total, because there are only
three available oval rugs. We can make a note of this three silk rugs.
rule: If three wool rugs are chosen, then we must choose
one to three silk rugs.
# Oval rugs = 2 or 3
We could draw these possibilities using columns:
Rule 2 tells us that we must select two or three wool Wool Silk
rugs:
2 2-3
# Wool rugs = 2 or 3 (not all 4) 3 1-3
If two oval rugs are selected, then there must be two
Rule 3, the last rule, is conditional. If the oval silk rug
to four rectangular rugs selected.
is selected, at least one rectangular silk rug is
selected. So if there is only one silk rug selected, it If three oval rugs are selected, then there must be one
must be rectangular. We draw the rule with an arrow to three rectangular rugs selected. If three oval rugs
diagram, along with its contrapositive: are selected, then one of the oval rugs is the silk rug,
which triggers Rule 3, and requires at least one
OS Æ 1 or 2 RS rectangular silk rug to be selected. We can draw this
out in our scratchwork as well:
No RS Æ no OS Oval Rectangular
Now we are ready for Step 4 of the Kaplan Method,
2 2-3
Key Deductions: We have only three oval rugs in our
3 1-3 (at least 1 RS)
collection, OW, OW, and OS. Since we must select A quick check to ensure that we have considered
either two or three oval rugs, our options are to select whether the rules created any Blocs (no), Limited
two oval wool rugs, or one oval wool and one oval silk Options (no), Established Entities (yes), Numbers
rug, or all three oval rugs. This means that we must issues (yes), or Duplication of Entities (no), and we are
ready to attack the questions.
31
PrepTest A Explained
Final Visualization: We need not consider Rule 2, because we are told that
# Oval rugs = 2 or 3 three wool rugs have been selected, thus fulfilling the
requirements of that rule.
# Wool rugs = 2 or 3 (not all 4) Rule 3 gets us our violator, and it’s answer choice (A),
one silk oval rug, along with our three wool rugs. Why?
OS Æ 1 or 2 RS
Because Rule 3 says that if the silk oval rug is
No RS Æ no OS selected, at least one silk rectangular rug must be
selected as well. Out of our remaining answer choices
Select 4 to 6: ((A), (D), and (E)), only (A) offers us the silk oval rug
without a silk rectangular rug.
OW OW RW RW RW OS RS RS
20. (E) “Could be true EXCEPT”
Wool Silk
2 2-3 Where the correct answer choice must be false in a
3 1-3 “not-if” question, use your deductions to spot the odd
man out.
Oval Rectangular
A quick comparison of the answer choices to our final
2 2-3
visualization reveals no reason why answer choices
3 1-3 (at least 1 RS) (A), (B), (C), and (D) could not be true. However, we
have already deduced that all three wool rectangular
rugs cannot be selected: (E). If we tried to select all
18. (D) Acceptability three of them, we would not be able to comply with
Rule 1, which requires us to choose at least two oval
Never pass up the point on an Acceptability rugs. Answer choice (E) therefore cannot be true, and
question—even if you haven’t had time to create a is our correct answer choice.
Master Sketch, you still can use the rules to
eliminate the wrong answer choices. 21. (B) “If” clause / “Must be true”
Rule 1 requires us to select at least two oval rugs.
Use the new “if” rule to draw deductions before
Neither (B) nor (E) includes two or three oval rugs, so
considering the answer choices.
we can eliminate them.
Rule 2 says that we may select only two or three wool If one silk rug is chosen, then we must choose three
rugs. Answer choice (C), with four wool rugs, is out. wool rugs in order to comply with the limitation
requiring at least four rugs to be selected for
Rule 3 tells us that if the oval silk rug is selected, we
exhibition.
must select at least one rectangular silk. Answer
choice (A) lists the oval silk rug, but no rectangular If only one silk rug is selected, then it cannot be the
silks, and so we eliminate it. oval silk, because Rule 3 requires a rectangular silk
rug to be chosen along with the oval silk rug. So the
That leaves (D) as the correct answer choice.
one silk rug is rectangular, which means that both oval
wool rugs must be chosen, in order to comply with Rule
19. (A) All acceptable “EXCEPT”
1. The final wool rug must be rectangular, because
Treat a “reverse” Acceptability question the same that’s all we have left. So here is how we would draw
way you treat an ordinary Acceptability question— these deductions:
use the rules to identify the one answer choice that
violates the rules.
OW OW RW RS
If three wool rugs are selected, those three rugs could
include two oval rugs required by Rule 1, so it is not
RW RW OS RS
necessary for the silk oval rug to be selected. That’s
why both (B) and (C) describe possible selections. Now we can just compare the answer choices to our
new sketch, to find the one that matches. Answer
choice (B) matches exactly. If there is only one silk rug,
32
Section 3: Logic Games
then the other rugs that are chosen must include two Rule 2 requires us to select two or three wool rugs. But
oval rugs and one rectangular rug. answer choice (A) adds only one wool rug to our roster.
We have found the rule-breaker, and we can move on,
22. (C) “If” clause / Acceptability knowing that each of the remaining answer choices
provides yet another possible selection of rugs that
Watch for unusual ways the LSAT writers can ask an complies with our rules.
ordinary question.
At first glance, this question looks like an “if” / “could 24. (C) “If” clause / “Must be true”
be true” question, but a closer look reveals that this is
Always take a moment to characterize the correct
nothing more complicated than an Acceptability
and incorrect answer choices before you work
question. The “if” simply tells us that we are at the low
through the question.
end of the number of rugs that must be selected. Each
answer choice is a proposed roster of rugs to exhibit, If the correct answer choice “must be true,” then the
and so we can simply use the rules to eliminate the wrong answer choices either “could be true” or “must
wrong answer choices. be false.”
Rule 1 requires us to select at least two oval rugs. Our evaluation of the numerical possibilities at the
Eliminate answer choices (A) and (E). outset of this game tells us that if all six rugs are
Rule 2 tells us that we can select no more than three chosen, three must be wool, and all three silk rugs are
wool rugs. It’s a little tricky to apply here, but if we look chosen. Since both rectangular silk rugs must be
at answer choice (B), we can see that the rule is chosen, (C) is our correct answer. For the record,
violated. If we have no rectangular silk rugs, then we (A) violates the rule that we must have at least two
cannot choose the oval silk rug. But without the oval oval rugs.
silk rug, we must choose the two oval wool rugs. Two (B) is possible, but not necessary, since we could
oval wool rugs plus two rectangular wool rugs makes select two oval rugs (one silk and one wool) and four
four wool rugs altogether, a clear violation of Rule 2. rectangular rugs.
Rule 3 tells us that selection of the oval silk rug requires (D) could be true, but is not necessarily true, as we
the selection of at least one rectangular silk rug. Both saw with answer choice (B).
(C) and (D) start off with all three oval rugs. This means
(E) is not possible under any circumstances, as we
that the oval silk rug has been selected. Answer choice
would not be able to comply with Rule 1 were we to
(D), however, has no rectangular silk rug, so it can be
choose all three rectangular wool rugs.
eliminated, leaving (C) as the good answer.
33
PrepTest A Explained
34
Section 4: Logical Reasoning
primarily to help the snake avoid becoming some other conclusions based on evidence concerning the distant
predator’s dinner. past, but rather, she is concerned with the evidence for
One way to determine whether or not the pits related one specific belief about the past.
more to predatory or defensive behavior would be to (C) Another mischaracterization of the argument,
compare the behavior of pit vipers to that of their couched in attractive, legal-sounding words. The
relatives without pits. If the pits in fact played a role in argument is not about an “absurd” comparison, but
the pit vipers’ defensive behavior, then one would about a questionable belief.
expect pit vipers to behave differently than other vipers (E) Yet another mischaracterization of the argument.
without pits when it comes to defensive behavior, and There is no “general principle” being applied here—the
one would expect pit vipers to have no significant argument and evidence is related to two specific
differences in predatory behavior, as compared to their aspects (the persecution of minorities as a reflection of
relatives without pits. (B) says just that, and popular hostility toward those minorities, and the
strengthens the researcher’s hypothesis. supposed reluctance of the medieval authorities to
(A) The argument makes no distinction between male engage in that persecution) of one specific historical
or female pit vipers, and so this answer choice is phenomenon (medieval persecution of minorities).
outside the scope of the argument.
(C) The argument is strictly about the role played by 5. (A) Strengthen the Argument
the infrared sensors possessed by pit vipers.
Watch out when a stimulus contains multiple points
Information about the chemical composition of vipers’
of view.
venom (or other distinguishing characteristics) is
outside the scope of and irrelevant to this argument. The stimulus actually contains two arguments: the
(D) Additional characteristics of pit vipers (such as a first, that official medieval persecutions of various
well-developed sense of smell) that may provide minority groups were done reluctantly and in response
information in addition to the thermal impressions they to popular hostility against minorities, and the second,
receive from their infrared sensors says nothing about that the first argument is “highly questionable.” Here,
the role played by those infrared sensors. This answer we are asked to strengthen the latter argument.
choice is also outside the scope of the argument. (A) strengthens the claim that the popular belief is
(E) Information about other defense mechanisms questionable. Evidence that the authorities themselves
possessed by some pit vipers says nothing about stirred up popular feeling against minorities makes it
whether the infrared sensors that distinguish pit vipers less likely that the authorities were “reluctant” to
from other vipers are also defense mechanisms. engage in the persecution, and also tends to disprove
Another answer choice that is outside the scope of the the claim that they were merely responding to popular
argument. sentiment against minorities. Good answer choice.
(B) If “official institutions” are the same thing as
4. (D) Method of Argument “medieval authorities,” that would weaken the
argument that we are trying to strengthen. If “official
Use Keywords to help characterize the pieces of an institutions” are not the same as “medieval
argument. authorities,” this answer choice is outside the scope of
The author notes a common belief among scholars, our argument.
and then opines that the belief is highly questionable. (C) If anything, this answer choice would support the
We then get two pieces of evidence, signaled by claim that at least some official persecution was in
keywords “for one thing,” and “for another.” The first keeping with popular hostility toward some groups. We
piece of evidence is that there is little evidence to want to support the contrary argument.
support the belief, and the second piece of evidence is (D) Evidence that many medieval authorities took
an alternate explanation for the observed phenomena. action to protect certain minorities may or may not be
That’s answer choice (D). relevant to this argument. Even if some minority groups
(A) mischaracterizes the argument. The argument is were under official protection, that says nothing about
that the prevailing view is incorrect. the authorities’ participation in the persecution of
(B) This one is far too broad given the scope of the other minority groups.
argument. The author is not concerned about all
35
PrepTest A Explained
(E) Punishment that does not rise to the level of music in the morning. Later that day, the people
persecution is beyond the scope of this argument. Our listened to the music again, as well as a second piece
only concern here is whether the medieval authorities’ of music. A strong majority preferred the piece they’d
persecution of certain minorities was reluctant or heard twice.
reflected popular sentiment against minorities. Answer choice (D) offers a complementary experiment:
the same procedure was followed, except this time, the
6. (C) Inference second piece was played twice. A substantial majority
of the listeners preferred the second piece (that they’d
The right answer choice for an Inference question
heard twice over the course of the day) in the new
must be true.
experiment. Neither experiment is particularly strong
The stimulus tells us that the frozen man lived at least evidence to support the conclusion (we don’t know
4,000 years ago (based on artifacts found with his whether the music pieces were equally attractive, or
body), and the nature of the glacier indicates that the whether the listeners for each experiment were from a
man died on the spot where his body was discovered. comparable pool of research subjects, for example),
We are also told that had the glacier not frozen the but the same strong preference for the piece that was
body shortly after the man died, or had the glacier heard twice during the course of the day, regardless of
thawed before this discovery, the body would not have which piece was the one that was played twice, is
been preserved. Put it all together, and that glacier another piece of evidence that makes the conclusion
must be at least as old as the body, that’s at least somewhat more plausible.
4,000 years old. (C) (A) weakens the argument. It describes an experiment
(A) The stimulus says that the artifacts found on the where there was no group preference for the musical
body are at least 4,000 years old. That is entirely piece heard twice.
consistent with some of the artifacts being more than (B) suggests that the piece called “Study I” was
4,000 years old. inherently more likable than “Study II,” and weakens
(B) The stimulus makes no mention of the man’s the argument.
cause of death, so we cannot rule out the possibility (C) The inclusion of professional music critics might
that he froze to death shortly before the glacier give a reason other than that proposed by the
encased his body. hypothesis to explain the group’s preference, which
(D) The fact that the body would not have been would weaken the argument. Moreover, all we know is
preserved had the glacier thawed prior to the present that at least one (“some”) of the listeners was a
time is good enough to guarantee that the glacier is at professional music critic. Unless a significant number
least 4,000 years old. It is not necessary for the of the listeners were professional music critics, this
artifacts to be just as perishable as the man’s body. piece of information has no effect on the weight of the
(E) While this answer choice may be true in the real argument.
world, nothing in the stimulus requires global warming (E) Another weakener—if a substantial number of
as compared to 4,000 years ago. So long as the people who listened to “Study I” in the morning were
climate in the area around the glacier did not warm not able to hear well, they would not necessarily be
sufficiently to thaw the glacier before the discovery of familiar with the piece when it was played for them a
the body, it does not matter whether the average global second time.
climate is warmer or cooler or the same as it was
4,000 years ago. 8. (E) Assumption
7. (D) Strengthen the Argument Subtle scope shifts can be the key to unlocking an
argument’s central assumption.
The right answer for a Strengthen question will not
After a natural disaster, the price of plywood often
necessarily make the argument iron-clad. A
increases above the price retailers charged for plywood
consistent piece of evidence can be a strengthener.
before the disaster. The author argues that even
As proof of the hypothesis that people prefer music though the price of plywood has increased, the
they have heard on an earlier occasion to music they retailers make no more money per sheet of plywood
are hearing for the first time, the author notes an than they did before the disaster. As proof, the author
experiment where 100 people listened to a piece of notes that after a disaster, the retailers’ cost to obtain
36
Section 4: Logical Reasoning
plywood frequently increases. The scope shifts from in the argument. The claim that the plywood’s cost to
the cost of plywood to the retailers’ profit per sheet of retailers increases after a natural disaster supports
plywood. For the conclusion to be true, the price the author’s main claim, that retailers aren’t making
increase for the consumer must be no more than the extra profit off of plywood following a natural disaster.
cost increase to the retailer. (D) The author never claims that the opposing view
Answer choice (E) says just that. Apply our Kaplan (that plywood retailers make undue profits following a
Denial Test to this answer choice, and we can confirm natural disaster) is implausible. He just argues that the
that it’s the right answer: if the price increase were opposing view is wrong.
higher than the increased cost to the retailers, they (E) The claim that the cost of plywood for retailers is
would in fact be profiting from the disaster, and the higher following a natural disaster is integral to the
argument falls apart. argument that the retailers do not make undue profit
(A) may very well be true, but does not affect the when they increase the price of plywood. The claim is
validity of the argument. The ability of people to pay the not mere background information.
increased prices after a disaster is outside the scope
of the argument. 10 (D) Weaken the Argument
(B) Whether or not the retailers pay the entire increased
When an argument is based on the results of an
cost to transport plywood says nothing about whether
experiment, evidence that the experiment is
or not they are charging their customers an amount
unrepresentative of actual conditions will weaken the
greater than the increased costs that they do pay.
argument.
(C) The author is not suggesting that retailers absorb
any of the increased costs—he is concerned with Based on experiment in which smoke samples from a
whether or not the retailers are increasing the price of forest fire were bottled and then analyzed to reveal
plywood above the amount their costs have increased. high concentrations of nitrous oxide, industrial
scientists (no bias there!) hypothesize that much of the
(D) If the amount of plywood shipped to an area
nitrous oxide that contributes to air pollution comes
following a natural disaster decreases from the amount
from forest fires. Answer choice (D) suggests that
shipped prior to the disaster, a tighter supply of
perhaps the process of bottling the smoke samples
plywood might contribute to a price increase. However,
has artificially raised the concentration of nitrous oxide
if this were occurring, then something other than an
within those samples, clearly weakening the
increase in the retailers’ costs would be driving up the
hypothesis.
price of plywood, and the retailers would, in effect, be
profiting from the disaster. This is a 180. (A) The identification of other potential sources for up
to 10 percent of the nitrous oxide present in the
9. (C) Role of a Statement atmosphere says nothing about whether or not forest
fires are responsible for the majority of the nitrous
A subsidiary conclusion is supported by evidence, but oxide present in the atmosphere.
also supports the author’s main conclusion. (B) The fact that many of the “pollutants” (not just
The clause mentioned in the question stem is nitrous oxide) present in our air come from the burning
supported by evidence (shipping costs for the retailer of fossil fuels (which themselves derive from
increase after a natural disaster), but the clause also wholesome organic matter) says nothing about whether
supports the main conclusion of the argument, that or not forest fires are a major source of nitrous oxide
retailers don’t make more per sheet of plywood than in the atmosphere.
they did prior to the natural disaster. That makes the (C) If anything, the existence of soil bacteria that feed
clause a subsidiary conclusion, answer choice (C). on ash and excrete small amounts of nitrous oxide into
(A) A “counterexample” is an example that serves to the atmosphere strengthens the argument by
disprove a claim. There is no claim in this argument suggesting another way that forest fires can contribute
that is contrary to the clause at issue—nothing in the to the amounts of nitrous oxide in the air.
argument suggests that costs don’t increase following
a natural disaster.
(B) A main conclusion is a claim that is supported by
evidence, but does not in turn support any other claim
37
PrepTest A Explained
(E) The existence of a different way of analyzing the points to a very limited number of theaters actually
forest fire smoke has no effect on the argument, seeing profits from successful films, which would
unless we also know the results of that alternative way explain the discrepancy.
of analyzing the smoke. We have no way of knowing (A) points to a reason why theaters in general might
whether on-site analysis of forest fire smoke would see lower profits, but doesn’t explain how box office
strengthen or weaken the industrial scientists’ receipts could have increased by 40 percent despite
hypothesis. this increase in movie-making costs.
(B) Falling ticket prices at “some” theaters might
11. (B) Logical Flaw
explain “some” of the bankruptcies, but standing
Arguments based on correlations are loaded with alone, provides no explanation for the increase in box
potential flaws. office receipts.
(D) Increased film advertising paid for by entities other
This argument presents a chain of correlations: the
than theater owners might explain the increased box
size and shape of a plant’s leaves are correlated to
office receipts, but makes the increase in bankruptcies
(dependent on) the climate in which the plant grew.
even more mystifying.
Likewise, climate is correlated to (dependent on)
altitude. The open question is how tight are those (E) tells us that those theaters that experienced
correlations? Altitude may determine climate, but over increased box office receipts also saw an increase in
what range of altitudes can one see the same climate snack bar profits as well, but that just deepens the
patterns? Nothing in the stimulus tells us. That is mystery relating to those theaters that went bankrupt.
precisely the problem identified in answer choice (B).
13. (E) Principle
(A) The ability of plant species to survive violent
changes in environment is outside the scope of the The principle that will justify a particular argument
argument. must match the evidence and conclusion exactly.
(C) The argument only requires the relation between
Mr. Smith’s daughter withholds consent for a kidney
leaf size and shape to be correlated to climate, not for
transplant that is urgently needed by the comatose Mr.
that relation to be the only physical characteristic of
Smith, on the ground that her father strongly objected
the leaves that is determined by climate. If the
to transplantation of organs from live donors, even
intensity of leaf color, for example, were also
though Mr. Smith’s objection is based on an incorrect
determined by climate, it would not affect the
belief about the dangers of organ donation to a live
argument in any way.
donor. The principle set forth in answer choice (E)
(D) The author does not rely on any supposed requires the daughter to withhold consent.
“analogy” between leaves and fossilized leaves—there
(A) would require the daughter to ignore her father’s
is no reason to believe that the fossilized leaves are
preferences and instead consent to the surgery to try
somehow unrepresentative of the size and shape of
to save her father’s life, unless he had prohibited her
the actual plant leaves.
from doing otherwise. The stimulus is silent on the
(E) The conclusion of the argument relates to the issue of prohibition, so we don’t know what this
place where the plant grew, not where the leaf principle would require the daughter to decide.
fossilized. That the leaf may have fossilized some
(B) clearly would require the daughter to consent to the
distance away from where the plant grew would have
kidney transplant surgery regardless of her father’s
no effect on the argument.
beliefs.
12. (C) Paradox (C) The stimulus is not explicit in this regard, but does
seem to suggest that the “urgently needed” transplant
Identifying the apparent contradiction makes it easier would in fact prolong Mr. Smith’s life. In that case, the
to find the right answer for a Paradox question. daughter would not be relieved of her obligation to
consent to the surgery for her father. At best (from the
How could there be a doubling of the number of movie
point of view of this answer choice), there’s no
theaters going bankrupt in a year where box office
evidence whether the surger y would prolong Mr.
receipts increased by 40 percent? Clearly, the
Smith’s life, in which case the principle would give no
theaters that are declaring bankruptcy are not the ones
guidance whatsoever to the daughter.
seeing the increased receipts. Answer choice (C)
38
Section 4: Logical Reasoning
(D) The stimulus expressly states that Mr. Smith’s 15. (E) Role of a Statement
belief that donating an organ would cause serious
disability to the donor is mistaken. The sufficient Use Keywords to assist you in identifying the role of
condition of this principle therefore is not fulfilled, and a particular claim within an argument.
so this principle would not justify the daughter’s Here, the claim at issue is prefaced with the word
decision. “although.” We know from Reading Comprehension
that “although” signifies an acknowledgement of a
14. (A) Parallel Reasoning (Flaw) potential objection or weakness in an argument that
will be overcome. That’s just what answer choice (E)
The conclusion test can be used to eliminate likely
describes.
wrong answer choices on Parallel Flaw as well as
ordinary Parallel Reasoning questions. (A) Evidence supports a conclusion, but the claim at
issue here, if true, potentially undermines the letter
The letter-writer makes the classic flaw of assuming writer’s argument against the editorial stating that
that something that is true of a group as a whole will burglars run almost no risk of being caught.
be true of any subgroup. Here, the group is “people
(B) This argument has a subsidiary conclusion that we
who commit crimes,” and the writer notes that at least
considered in Question 14. The author provides no
70 percent of these people are caught. The writer than
suppor t for the claim that some burglars will
contends that it must also be true that a large
undoubtedly escape, and so it cannot be either a
percentage of burglars (a subgroup of “criminals”) will
subsidiary or main conclusion of this argument.
eventually be caught.
(C) The author is most concerned with proving that
Of all the answer choices, only (A) concludes that “a
burglars do face a significant risk of being caught, and
large percentage” of a subgroup (this time it’s nurses,
so burglars who escape capture do not constitute an
who are a subgroup of the group, “professional
example of the kind of case under discussion.
persons”) have a particular characteristic (being self-
employed). Look more closely, and we see that exactly (D) The claim that some burglars will escape is not
the same classic flaw is at work in (A): we have a another way of saying that someone who commits a
group (professional persons) of whom “a large burglary runs almost no risk of being caught.
percentage” is self-employed. The speaker then
assumes that this is also true of a subgroup, nurses. 16. (B) Inference
For the record:
The right answer choice for an Inference question will
(B) This argument draws a conclusion about “some frequently involve the combination of two or more
people,” as opposed to a “large percentage” of a claims made in the stimulus.
subgroup. Moreover, the argument is not flawed, let
alone flawed in the same way as the stimulus. In this stimulus, we learn that just about any chemical
is carcinogenic, if given in doses large enough to kill
(C) This argument draws a conclusion about “all”
cells. We also learn that until now, studies of the
professional persons who have changed their careers,
carcinogenicity of food additives were performed by
and is logically valid.
administering to test animals doses of the additives
(D) This argument draws a conclusion about “many” that were not only larger than the largest possible
doctors. Although the “large percentage” of doctors human exposure levels, but doses that were large
described in the evidence for this argument can enough to kill many cells in the test animals’ bodies.
coincide in meaning with “many” (both are vague terms Combine those two claims, and it must be true that
that are best translated as “some”), the flaw of the until quite recently, carcinogenicity studies of food
argument in the stimulus does not appear in this additives have overestimated the carcinogenicity of
argument. The conclusion of the argument here is those substances, (B).
about “many” members of the original group, not the
(A) is a 180. The stimulus specifically states that
subgroup, and the argument is logically valid.
previous carcinogenicity studies of food additives
(E) This argument, like the one in (D), draws a conclusion used doses large enough to cause cell death in the
about “many” engineers (as opposed to the subgroup of test animals.
engineers who are also managers). The formal logic
(C) may very well be true in the real world. However, the
claims in this argument can easily be combined, and the
stimulus does not provide us with any information that
argument’s conclusion is logically valid.
39
PrepTest A Explained
would allow us to distinguish between truly carcinogenic 18. (D) Point at Issue
substances and those that are not truly carcinogenic.
Sometimes the point at issue is pretty obvious.
(D) This answer choice may also be true in the real
world (perhaps depending upon where one lives), but Efraim argues that because attempts to follow advice
there is no support for it in the stimulus. from the popular press about the avoidance of health
(E) goes beyond the information of the stimulus. While risks make people anxious, and anxiety is itself a major
the stimulus clearly tells us that the way health risk, people should disregard health advice in
carcinogenicity studies of food additives have been the popular press. Felicity clearly states her
conducted likely overstates the carcinogenicity of some disagreement with Efraim (“you are wrong”), and
food additives, we cannot tell whether this is true of provides the example of how articles in the popular
“most” food additives that have already been banned press helped people learn about the risks of smoking,
due to concerns about carcinogenicity. which in turn led many people to improve their health
by quitting smoking.
17. (B) Assumption (D) sums up the dispute quite nicely, and we can use
the Kaplan Decision Tree to confirm it’s the right
When an argument concludes that there is only one way answer: both Efraim and Felicity have an opinion about
to achieve a particular result, the argument assumes whether people can improve their health by heeding
that there are no other ways to achieve that result. advice given in the popular press. Efraim is committed
The author concludes that if highway tolls are to a negative view, while Felicity is committed to a
eliminated, the entire cost of maintaining the highways positive view of the issue. We have identified the point
(which had previously been covered by the tolls), would at issue in their argument.
have to be paid for by an increase in general taxes. The (A) Efraim implicitly states that the popular press
author assumes that there is no other way to pay for describes actual health risks when he acknowledges
any portion of these maintenance costs. Answer choice that his advice to ignore the ar ticles seems
(B) strikes at the heart of the assumption, by “paradoxical.” Felicity provides the example of articles
eliminating one possible alternate source of funds for about the health risks posed by smoking,
highway maintenance. acknowledging that at least sometimes, the popular
Apply the Kaplan Denial Test, and we can see that this press describes actual health risks.
must be the assumption: if savings from eliminating (B) Efraim mentions something about the articles
the collection of tolls could be used to pay part of the causing anxiety, but never makes the comparison
highway maintenance expenses, then it is not true that contained in the answer choice. Felicity never
the maintenance expenses would need to be paid discusses any relationship between anxiety and
entirely out of an increase in general taxes. attempts to reduce health risks. This cannot be a point
(A) The argument says nothing about the procedure for at issue.
authorizing highway maintenance; this answer choice is (C) Both Efraim and Felicity could agree that there are
outside the scope of the argument. some people who ignore journalists’ advice about
(C) How the maintenance money is to be spent avoiding health risks.
(whether on preventive maintenance or on repairs) is (E) Efraim implies that the only way to avoid anxiety
outside the scope of this argument. due to attempts to avoid health risks is to disregard
(D) When we see a comparison in an answer choice, as the articles. Felicity never discusses anxiety.
we see here, our first reaction should be to consider
whether the comparison is relevant to the argument. 19. (D) Assumption
The stimulus says nothing about how the existence of
Identifying the type of conclusion in an argument can
highway tolls as a funding source affects the total cost
be useful in identifying the evidence that is missing.
of highway maintenance, and so this is an irrelevant
comparison. Efraim’s conclusion compares the health risks of
(E) The author never considers other reasons why ignoring journalists’ advice to the health risk of trying
general taxes might be increased, and never suggests to follow that advice. His evidence, however, contains
that eliminating highway tolls is the only possible no comparison of the relative risks posed by anxiety
reason why general taxes could be increased. versus the health risks described in the articles. The
assumption is a missing piece of evidence that will fill
40
Section 4: Logical Reasoning
in the gap between evidence and conclusion. Here, we (C) The problem with the argument is that it does
need some kind of evidence comparing the relative assume that a characteristic (insect-attracting
risks of attempting to follow versus ignoring the advice features) is unique to the class of orchids that are
at issue, and (D) is the only answer choice that gives pollinated solely by insects.
us just that. (E) The argument is not based on a generalization
(A) Even if a significant proportion of people who try to based on what is known about one member of an
follow journalists’ health advice are doomed to be unrelated class. Rather, the author is trying to argue
unsuccessful in completely avoiding specific health that one member that possesses a characteristic
risks, that says nothing about the relative harm posed known to be possessed by all members of a certain
by anxiety versus the health risks. This one doesn’t fill class must belong to that class.
the gap between evidence and conclusion.
(B) Evidence about how the journalists’ advice causes 21. (A) Method of Argument
anxiety says nothing about the relative health risks
The correct answer for a Method of Argument
caused by either trying to follow or disregarding
question will describe exactly what happens in the
journalists’ health advice.
stimulus argument.
(C) The lack of unanimity in journalists’ recommendations
for ways to reduce particular health risks may explain how The author argues that the failure of Portuguese
attempting to follow their advice causes anxiety, but says contemporaries to mention the fact that King
nothing about the relative health risks due to either Sebastian was killed in a battle on Moroccan soil is
following or ignoring the journalists’ advice. because Sebastian’s death was so humiliating that
they could not bring themselves to write about it. The
(E) Evidence that suggests that anxiety is hard to treat
evidence in support of this argument? A series of
doesn’t say anything about whether anxiety due to
reasons why the omission was not due to ignorance of
trying to follow journalists’ health advice poses more of
King Sebastian’s death. None of the evidence
a health risk than disregarding the health articles.
discusses the psychological cause for the omission set
forth in the conclusion; the author merely eliminates an
20. (D) Logical Flaw
alternate explanation for the omission as her proof.
The LSAT writers have many ways of expressing the That’s answer choice (A).
logical flaw of confusing what is sufficient with what (B) There is no dispute in this argument over whether
is necessary. or not King Sebastian actually died in the battle.
The stimulus tells us that if an orchid species is (C) There is no contradiction, apparent or hidden, in
pollinated solely by insects, then it has features that the argument’s premises (evidence). All of the
attract insects. The argument concludes by stating that evidence points to the conclusion that the writers who
an orchid that possesses features that attract insects failed to mention King Sebastian’s death were not
must be pollinated solely by insects, a classic reversal ignorant of the fact that he had died in the battle.
of sufficient and necessary conditions. (D) No general psychological principle is applied in the
(D) is one way to express that concept: the argument argument, although the author describes the purported
treats a characteristic (insect-attracting features) cause for the omission as “psychological” in nature.
known to be true of one class of things (orchids that (E) The author offers grounds for doubting the
are pollinated solely by insects) as if that characteristic reliability of historical reports that fail to mention the
were unique to that class (assuming that if an orchid death of King Sebastian, but those reports do not
has insect-attracting features, then it must be conflict with the author’s main conclusion. To the
pollinated solely by insects). contrary, the argument’s main conclusion attempts to
(A) The argument does not attempt to prove that two explain why the historical reports omitted mention of a
kinds of orchid species have only one characteristic in presumably important fact.
common.
(B) The argument is not about two classes of things,
nor is it about there being only one characteristic that
is different between two classes of things.
41
PrepTest A Explained
Critical reading skills are especially important in A parallel argument must have the same number of
Inference questions. components (pieces of evidence plus the conclusion)
as the stimulus argument.
Based on the stimulus, we know that a particular
species of thrips can reproduce either by laying eggs or The stimulus argument provides two pieces of
by bearing live young, although any particular brood of evidence, which logically support its conclusion. We
young will be either hatched from eggs or born live. All can eliminate answer choices where the conclusion
offspring hatched from eggs are female, and all rests on only one piece of evidence.
offspring born live are male. Any particular live-born (A) This argument is based on only one piece of
brood will be smaller than any particular brood hatched evidence. Eliminate it.
from eggs. Thus, we can conclude that given equal
(B) This argument is based on only one piece of
numbers of broods, there will be more female than
evidence. Eliminate it.
male offspring.
(C) This argument is based on two pieces of evidence.
The author, however, is not done with us. He continues,
However, unlike the stimulus argument, which is
telling us that a larger proportion of male (remember,
logically valid, the conclusion of this argument does not
that’s the live-born offspring) than female offspring
necessarily follow from the evidence. Eliminate it.
(hatched from eggs) survive to adulthood. Interestingly
enough, among these thrips reaching adulthood, the (D) This argument is based on two pieces of evidence.
proportion of males to females is about equal. The evidence, taken together, logically supports the
conclusion. Just like in the stimulus, we need to match
The best way to handle such a long, complicated
up two sets of entities. Just like in the stimulus, there
stimulus is to tackle each answer choice in order, and
is more of one type of entity than the other. Just like in
ask whether or not that answer choice is a valid
the stimulus, this argument concludes that some
inference (must be true).
members of the smaller entity set will be matched with
(A) The thrips are the first species found that can more than one member of the larger entity set. This is
reproduce by either egg-laying or live birth. That doesn’t the parallel argument.
necessarily mean that they are the only species
(E) This argument is based on only one piece of
capable of doing so.
evidence. It can’t be the parallel argument.
(B) The stimulus says that any particular female will
use only one method of reproducing per brood, but 24. (D) Inference
there is no information about whether individual
females who reproduce in one way are or are not Sometimes skills from Logic Games can be applied to
capable of reproducing in the other way. a Logical Reasoning question.
(C) If a greater proportion of live born (male) thrips This question tests your loose sequencing skills. We
reach adulthood than the propor tion of hatched have a series of statements comparing the size of
(female) thrips, but the proportion of male to female offices on the first through fourth floors of a building.
adult thrips is about equal, it must be the case that We can treat the stimulus like a Loose Sequencing
more thrips are hatched than are born live. (C) must be Logic Game. We could diagram the rules just as we
true based on our stimulus, and it is the correct would for a Logic Game, going from largest to smallest
answer choice. For the record: (read left to right):
(D) It’s possible that there could be equal numbers of 4th floor \ / 1st floor
male and female broods, with fewer males being born
2nd floor
than females are hatched, but the greater proportion of
rd
Most of the 3 floor / \ Some 3rd floor
males surviving to adulthood then balances out the
proportion of male to female adult thrips. However, this We can compare each answer choice to our sketch to
need not be true. see what must be true.
(E) It’s possible that there are some females who use (A) If all first floor offices are smaller than all second
only one of the two methods of reproduction over the floor offices, and all second floor offices are smaller
course of their lives, but this need not be true. than all fourth floor offices, no first floor offices can be
42
Section 4: Logical Reasoning
43
1-800-KAP-TEST | kaptest.com
*LSAT is a registered trademark of the Law School Admission Council. Printed in USA ©2006 Kaplan, Inc.